SAT阅读必备OG之黑人女性
SAT阅读高分攻略系列(四--六)
SAT阅读高分攻略系列(四)例B.OG P799,对象Black women黑人和女人作为美为社会长久以来的弱势群体。
一直属于SAT阅读的高频写作对象。
这位作者关注的对象却是黑女人,自然大受College Board青睐。
这类文章的个非常好的特点:作者对其写作对象的感情必然是正面的。
毕竟人家受歧视这么多年才终于翻身,College Board若是收录依旧偏激的文章岂不是要引起公愤?故而有关此类人群的文章要么是赞扬他们无畏的奋斗,要么是悲叹他们不幸的遭遇,要么是分析社会现状并提出改良建议,中心都很好把握。
斜体介绍的即是写作对象,照例按首末句跳读文章。
不难发现它的结构与前面的例子相似:开头提出了一个社会现象,预计是作者要批判的,果然到第四段“sad”,“unnecessary”表明了作者态度,末段举了个例子,末句明确点出了思想感情。
从中我们可以总结一下社会评论类文章一副常用的框架——开篇介绍历史背景或事实,往往是客观的描述,但可以从语气中找到作者态度,这态度多半是反对的;随后分析这样现状的产生原因,用到的则是作者自己的观点和事例,主观地表明作者的观点;最后可能提些建议或作出对未来的假设,也可能话锋一转从一定程度上肯定原来仅驳的观点。
在这样的框架中作者观点通常在文章中后部出现,并且在结尾处最容易找到(如此文,另见DG P648、P619),但当结尾出现明显转折,让步或举例时则很可能不是作者观点。
了解文章结构思路对快速把握中心很有好处。
回到例文我们来看几道题目。
另10题“the purpose”,又可以利用作者观点来做。
题中指出的line4~10即是说到黑人女性受到的对待,这是作者反对的,选项A、B、E与相关。
仔细再读选项,A、B中分别有“defend”、“discredit”,而很显然原文中作者并未在第一段流露明显感情倾向,只是较为客观地介绍了一个社会现象,与 E项中“note”更为相符。
从而我们遇到了第二条College Board原则:选项中暗含的态度非常重要。
SAT-OG易错题
OG Test 1Section 65.We generally think of Canada as the northern neighbor of the United States, and more thanhalf of the states extend farther north than Canada’s southernmost point.(A)States, and more than half of the states extend(B)States, and it is the case that more than half of the states extend(C)States, but more than half of the states extending(D)States, whereas more than half of the states are extending(E)States; however, more than half of the states extend6.The three volumes of memoirs by Wole Soyinka begin with his childhood in a Nigerianvillage and culminate with his years at the University of Ibadan, one of the best universities in West Africa.(A)begin with his childhood in a Nigerian village and culminate(B)that begin with his childhood in a Nigerian village and culminate(C)have begun with his childhood in a Nigerian village and culminating(D)beginning with his childhood in a Nigerian village and culminating(E)are begun as a child with in a Nigerian village and culminate7. Dressed in a crisp, clean uniform, it reflected the efficient manner of the tour guide as shedistributed maps for a walking tour of Canberra.(A)Dressed in a crisp, clean uniform, it reflected the efficient manner of the tour guide (B)Dressed in a crisp, clean uniform, the efficient manner of the tour guide was reflected (C)Dressed in a crisp, clean uniform that reflected the efficient manner of the tour guide (D)The crisp, clean uniform of the tour guide reflected her efficient manner(E)The crisp, clean uniform of the tour guide, a reflection of her efficient manner9. A review of the composer’s new symphony called it confusing because of its unusualstructure, and its melodious final movement makes it elegant.(A)structure, and its melodious final movement makes it elegant(B)structure, although elegant by having its melodious final movement(C)structure, and it is elegant with its melodious final movement(D)structure while having a melodious final movement makes it elegant(E)structure but elegant because of its melodious final movement11.The famous battle depicted in the film Braveheart took place in northern England, and manypeople assume that it was the Scottish Highlands.(A)and many people assume that it was(B)many people assuming(C)but many people assume it to be(D)not what many people assume(E)not, as many people assume, in13. From about A.D. 700 to 1600, sculptors created nearly 1,000 colossal rock statues on theA B Cremote and tiny Easter Island. No errorD E17. According to educational statistics, the average age of college students has risen quiteA B Cnoticeable over the past 25 years. No errorD E20. According to some demographers, the number of United States citizens aged 65 or older isA B Clikely to rise to 87 million by 2050. No errorD E25. As their brains mature neurologically, infants become more capable to distinguish the shapesA B Cand textures of the objects around them. No errorD E29.There is probably no story more dramatic than baseball’s great hitter and right fielder, HankA B C DAaron. No errorESection 102.The watercolors it has on display by the museum represent the era when Japan’s emergencefrom feudalism and isolation inspired its artists to explore new themes and techniques.(A)it has on display by the museum represent the era when(B)that it, the museum, is displaying represents the era of(C)on display at the museum represent the era when(D)displayed at the museum representing the era when(E)being displayed at the museum represents the era while3.The origins of the Teapot Dome scandal can be traced to the presidency of TheodoreRoosevelt, William Howard Taft, and Woodrow Wilson.(A)can be traced to the presidency of(B)can be traced to the presidencies of(C)happened in the presidency of(D)happening during the presidencies of(E)that happened in the presidency of9. In 1972, to reduce pollution in the Great Lakes, limits having been set by the United Statesand Canada on the amount of phosphorus that could be discharged into Lakes Erie and Ontario.(A)limits having been set by the United States and Canada(B)limits set by the United States and Canada(C)limits have been set by the United States and Canada(D)the United States and Canada have set limits(E)the United States and Canada set limits14. Acquaintances of Alexei have commented that he is at once annoying because of hisunpredictability but his imagination is still a delight.(A)but his imagination is still a delight(B)although he is delightfully imaginative(C)and he is delightful in his imagination too(D)while being imaginative and they are delighted(E)and delightful because of his imaginationOG Test 2Section 627.According to some theorist, what any particular bird can eat could change with even theA B C Dslightest variation in the shape of its back. No errorE23.At the conclusion of the novel The Great Gatsby, Nick Carraway, a young Midwesternerrecently arrived to New York, moodily watches the blinking green light at the tip of LongA B C DIsland. No errorESection 106.An artist who explores Mexican cultural themes, the art of Maria Elena is world renowned. (A)An artist who explores Mexican cultural themes, the art of Maria Elena is world renowned(B)To explore Mexican cultural themes, the work of artist Maria Elena is world renowned (C)Artist Maria Elena has explored Mexican cultural themes, the art of which is world renowned(D)An artist who has explored Mexican cultural themes, Maria Elena’s art is world renowned (E)Maria Elena is a world-renowned artist whose art explores Mexican cultural themes12.On October 13, 1955, at the Six Gallery in San Francisco, Allen Ginsberg read his poemHowl, being the inauguration of both a new style in poetry and the Beat movement.(A)Howl, being the inauguration of both(B)Howl, being inaugurated(C)Howl, it was the inauguration of both(D)Howl, whose inauguration of both(E)Howl, thus inaugurating both13.Indicating their desire to extend free enterprise Canadians elected a member of theProgressive Conservative Party, Kim Campbell, as Prime Minister in 1993.(A)Canadians elected a member of the Progressive Conservative Party, Kim Campbell, as Prime Minister(B)Canadians’ election of a member of the Progressive Conservative Party as Prime Minister was Kim Campbell(C)Kim Campbell of the Progressive Conservative Party was elected Prime Minister of Canada(D)the Progressive Conservative Party’s Kim Campbell was elected Prime Minister of Canadians(E) a member of the Progressive Conservative Party, Kim Campbell, was elected by Canadians as Prime MinisterOG Test 3Section 64.Nursing and physical therapy are an example of health-care fields that have shortages instaffing.(A)are an example of health-care fields that(B)are examples of health-care fields that(C)are examples where health-care fields(D)exemplifies a health-care field that(E)exemplify health-care fields where they6. The company maintains computer systems for small businesses, plus it will manage theirpayroll accounts.(A)businesses, plus it will manage(B)businesses, in addition it will manage(C)businesses and manages(D)businesses, and, additionally, they also manage(E)businesses, it manages10. To introduce itself to a wider audience, the little-known band sold its CD’s cheaply toenthusiastic fans, who in turn shared the music with friends.(A)sold its CD’s cheaply(B)sold their CD’s cheaply(C)sells its CD’s at a low price(D)prices their CD’s low for to sell(E)will sell its CD’s at a cheap price11. Twice as many bird species inhabit Ecuador as in North America.(A)as in(B)as inhabit(C)instead of in(D)when compared to(E)than21. Ms. Kovak proudly displayed her research group’s most ingenious invention, a vacuumA Bcleaner that empties its own dust bag when pressing a button. No errorC D22. Famous for their sticky feet, the gecko can run up walls and across ceilings as well as hangA B C Dfrom a surface by its toes. No errorE26. Traffic was heavy, so by the time Brianne finally arrived at the theater, we waited for her forA Ban hour, missing the entire first act of the play. No errorC D E28. Although familiar to us from representations in ancient art, war chariots are rare museumA Bartifacts because by the sixth century B.C. they were no longer used in battle. No errorC D E29. A volunteer organization, the Covington Soup Kitchen has been feeding needy families sinceA B1977, annually distributing nearly a million pounds of food each year. No errorC D ESection 105.The Poetry book Society is an organization in London that provides information and guidancefor all lovers of contemporary poetry and giving them discounts on books of poetry.(A)provides information and guidance for all lovers of contemporary poetry and giving them discounts on books of poetry(B)provides information, guidance, and the getting of discounts on books of poetry for all lovers of contemporary poetry(C)not only provides information and guidance for lovers of contemporary poetry but also gives them discounts on books of poetry(D)has been providing information and guidance for lovers of contemporary poetry and were giving discounts on books of poetry(E)are providing information and guidance for all lovers of contemporary poetry and having given them discounts on books of poetry11.Theodore Roosevelt was not just a great reformer; he was also a great president.(A)Theodore Roosevelt was not just a great reformer; he was also a great president(B)Theodore Roosevelt was not just a great reformer, and also a great president(C)Theodore Roosevelt was not just a great reformer; but also a great president(D)Great not just as a reformer, but Theodore Roosevelt was also a great president(E)Not just as a reformer, Theodore Roosevelt was also a great president13. In the novel, Jane Eyre must make many difficult choices, like when she forces herself toleave the house of Mr. Rochester, the married man she loves.(A)like when she forces(B)and, as an example, when she forces(C)for example, by forcing(D)as exemplified by when she forces(E)including forcingOG Test 4Section 73.The campus newspaper does not print as much world news as does my hometown.(A)as does my hometown(B)as does my hometown newspaper(C)compared to what my hometown does(D)like my hometown newspaper does(E)like the one in my hometown does11. Though heavily dependent on the government for business and information whileuniversities supply the space research center with talent, as a corporation it remains independent of both.(A)information while universities supply the space research center with talent, as a corporation it remains(B)information and on talent by universities, the space research center, a corporation(C)information and on universities supply for talent, the space research centre is a corporation (D)information, universities supply the space research center with talent, but it is a corporation16. Along the curve of island known as the Florida Keys lies a reef of living coral, the only oneA B Cof a kind in the continental United States. No errorD E18. Because he is absent when his rivals voted against his proposal, Selby is worried aboutA B C Dmissing future meetings of the board of directors. No errorE19.In those cities in which public transportation is adequate, fewer traffic problems occur andA Bpedestrians are rarely involved in accidents. No errorC D E24. Since some people are convinced that dowsing a method of finding underground water withA Ba Y-shaped stick, is effective, but others condemn the procedure as mere superstition. NoC D Eerror25. Intense preoccupation on technique appears to be the one trait that great pianists have inA B C Dcommon. No errorE27. Also supported by the commission was the proposed health clinics and the proposed centerA B Cto distribute information on job-training opportunities. No errorD ESection 104.Journalists should present a balanced view of the news but with their goal to stir discussionand unsettle complacent thinkers.(A)with their goal to stir(B)should also stir(C)aiming at the same time to stir(D)also trying to stir(E)its goal should also be in stirringcking good instruction, my mistakes in creating a graph to illustrate historical trends werenumerous.(A)my mistakes in creating a graph to illustrate historical trends were numerous(B)I made numerous mistakes in creating a graph to illustrate historical trends(C)there were numerous mistakes in creating a graph to illustrate historical trends(D)I created a graph to illustrate historical trends with numerous mistakes(E)the graph I made for illustrating historical trends had numerous mistakes12. By simply entering an Internet website or calling a toll-free number, a catalog order can beplaced for almost anything from cheesecakes to fully equipped desktop computers.(A)a catalog order can be placed(B)by placing a catalog order(C)they will place your catalog order(D)you can place a catalog order(E)your catalog order can be placedOG Test 5Section 64. The issue the council debated, which was whether repeal of rent control will improve housingor just increase profits for landlords.(A)debated, which was whether repeal of rent control will improve(B)debated was if they would repeal rent control would this improve(C)debated was the repeal of rent control would in improved(D)debated was will repealing rent control mean improvement in(E)debated was whether repeal of rent control would improve9. Although the English artist William Blake never having painted portraits, he regarded them asmerely mechanical reproductions that, despite their popularity, lacked true creativity.(A)Although the English artist William Blake never having painted portraits, he regarded them (B)The English artist William Blake never painted portraits, he regarded them(C)Never having painted a portraits, they were regarded by the English artist William Blake (D)The English artist William Blake never painted portraits, regarding them(E)The English artist William Blake never painted portraits because he regarded them22. Although the politician was initially very sensitive to be criticized by the press, he quicklyA B Cbecame more confident about responding to reporters’ sometimes pointed questions. No errorD E25. The decline in science education during the period had two causes: less funding for scientificA Bresearch with a decrease in jobs related to space and defense. No errorC D E26. The number of awards given this year to biochemists accentuate the significant gains beingA B Cmade in the study of the chemistry of living organisms. No errorD E28. Until it can be replaced by a faster, more efficient, and more economical means ofA Btransportation, trucks will carry most of the freight within and through metropolitan areas.C DNo errorE29. Nearly all of the editors of the magazine agree that of the two articles to be punished.A B CFujimura’s is the more exciting. No errorD ESection 104. No sooner had Andrea del Sarto traveled to France to work for the French king but his wifepersuaded him to return to Italy.(A)but his wife persuaded him to return(B)but his wife had him persuaded into returning(C)than he was persuaded by his wife that he will return(D)but he was persuaded by his wife into returning(E)than his wife persuaded him to return10. Until being widely hunted for its ivory and blubber in the eighteenth century, warlruses wereplentiful in the waters of the northeastern United States.(A)Until being widely hunted for its(B)Before having been widely hunted for its(C)Up to them being widely hunted for their(D)Until they were widely hunted for their(E)Before they have been widely hunted for their11. Jesse passed the California bar examination last year, and he has been practicing law inCalifornia ever since.(A)and he has been practicing law in California ever since(B)since that time he has practiced law there(C)where ever since he practices law(D)he has been practicing law in California since then(E)and since then is practicing law thereOG Test 6Section 616. The young fish were very tiny, yet each of them ate many times its own weight in solid foodA B C Devery day. No errorE24. The record left by fossils, the ancient remains of plants and animals, provide scientists withA Btheir primary source of information about prehistoric life. No errorC D E25. The exchange between the teacher and the student promotes learning far different from thatA Bwhich results as the student listens but does not participate. No errorC D E29. Available through the school’s guidance office is a job directory and a list of job referralAcenters that provide information for student in need of employment. No errorB C D ESection 102. The duties of a firefighter are often as dangerous as a police officer.(A)as a police officer(B)as those of a police officer(C)like a police officer(D)such as a police officer(E)like a police officer’s8. Having Florence Nightingale as their leader and a growing awareness of the need forcleanliness helped to bring about the establishment of standers of hygiene in hospitals.(A)Having Florence Nightingale as their leader(B)Having the leadership of Florence Nightingale’s(C)Florence Nightingale as their leader(D)To be led by Florence Nightingale(E)The leadership of Florence Nightingale9. An expedition was sent in 1949 to check a Turkish villager’s reporting he had seen theremains of Noah’s ark on Mount Ararat.(A)a Turkish villager’s reporting(B)the report of a Turkish villager(C)the reporting of a Turkish villager(D)that a Turkish villager who reported(E)the Turkish villager report sayingOG Test 7Section 45. Great literature, such as the Greek tragedies or the novels of Jane Austen, endures by theirspeaking directly and freshly to each new generation of readers.(A)by their speaking directly and freshly(B)by its speaking direct and fresh(C)because it speaks directly and freshly(D)because of speaking direct and fresh(E)since they speak with directness and freshness6. The services of architect I.M.Pei are always very much in demand considering that hisbuildings combine both beauty and an affordable price.(A)considering that his buildings combine both beauty and an affordable price(B)considering that his buildings combine both beauty and affordability(C)because his buildings combine beauty and affordability(D)because his buildings combine will combine not only beauty but also an affordable price (E)being that his buildings will combine both beauty and affordability7. The tragic story of Paolo and Francesca was the subject of a poem and a symphony, beingpopular throughout the nineteenth century.(A)The tragic story of Paolo and Francesca was the subject of a poem and a symphony, being popular throughout the nineteenth century(B)The tragic story of Paolo and Francesca was popular throughout the nineteenth century, where it is being made the subject of a poem and a symphony(C)The tragic story of Paolo and Francesca was popular throughout the nineteenth century, when it was the subject of both a poem and a symphony(D)Paolo and Francesca, whose tragic story was the subject of a popular poem and symphony throughout the nineteenth century(E)Being the subject of a poem and a symphony, the tragic story of Paolo and Francesca having been popular throughout the nineteenth century10. Just as Ireland has produced many famous writers and the Netherlands an abundance offamous painters, so Finland has provided a large number of famous architects.(A)so Finland has provided a large number of famous architects(B)Finland provides famous architects, and by large numbers(C)Finland’s contribution is to provide famous architects in a large number(D)and so then, for Finland, a large number of famous architects is provided(E)and like them Finland has provided a large number of famous architects11. Readers of the novice writer’s recent book have said that it is at once frustrating because ofits chaotic structure but its originality is still a delight.(A)but its originality is still a delight(B)although it is delightfully original(C)and it is delightful in its originality(D)while being so original as to delight them(E)and delightful because of its originality20. In 1850 Jim Beckwourth, a Black American explorer, discovered in the mountains of theA BSierra Nevada a pass soon becoming an important gateway to California gold-rush country.C DNo errorE22. The Mount Isa mine complex is one of the most highly mechanized in the world, plus beingA B Cthe largest single industrial enterprise in Queensland. No errorD E24. The warning in the plays is clear: unless we restore the integrity of the family, all traditionalA B Cvalues will disappear. No errorD E25. As a student becomes familiar with both early and contemporary Native American literature,Aone may notice that traditional stories have influenced recent ones. No errorB C D E26. John Edgar Wideman is regarded to be one of the most talented writers of the late twentiethA Bcentury and is often compared to such literary giants as Ralph Ellison and Richard Wright.C DNo errorESection 105. One result of the fire department’s new contract is that they can no longer offer overtime payon weekends.(A)One result of the fire department’s new contract is that they can no longer offer overtime pay on weekends.(B)As one result of it’s new contract, the fire department can no longer offer overtime pay on weekends.(C)One result of the fire department’s new contract is that overtime pay on weekends can no longer be offered to them.(D)The fire department’s new contract results in their no longer being able to offer them overtime pay on weekends.(E)One result of the fire department’s new contract are that offering overtime pay on weekends is no longer possible.10. Arelia believes that cloth draped over hills is an art form that cannot last long enough as awork of art either to please or influence future generations.(A)cannot last long enough as a work of art either to please or influence future generations (B)cannot last long enough as a work of art, for ages to come neither pleasing or influencing future generations(C)is not a lasting work of art that for ages to come will either please or influence future generations(D)is not a lasting work of art that will remain for ages pleasing and influencing future generations(E)will not last long enough either to please or to influence future generations11. The price of gold has been influenced by continued inflation and because people have lostfaith in the dollar.(A)because people have lost faith(B)because of the loss of faith(C)by people which have lost faith(D)losing faith(E)loss of faithOG Test 8Section 45. In response to the traditional assumption that all readers are basically alike, feminist criticshave emphasized that every woman reads from their own unique perspectives.(A)that every woman reads from their own unique perspectives(B)that every woman reads from her own unique perspectives(C)that all women reading from a unique perspective of their own(D)how women reading each have unique perspectives(E)how the unique perspectives of women are in their readings9. Today the primary role of advertising may be to appeal and persuade rather that what it oncedid, educating and informing.(A)what it once did, educating and informing(B)what it once did, which was educating and informing(C)what it once did, education and information(D)educating and informing, what it once did(E)what it once was, to educate and inform20. The use of irrigation in the once-arid region have increased the production of alfalfa and ofA B Cmany other crops as well. No errorD E21. Unfortunately, the opening of the new library complex, previously scheduled for nextA BSeptember, would be delayed for several months because of construction difficulties.C DNo errorE23. Given her strong sense of social justice, Burns vehemently protested over her party’s failureA B Cto support a tax decrease for senior citizens. No errorD ESection 108. The practice of renaming a street Martin Luther King Boulevard has been adopted throughmany cities in honoring the civil rights leader.(A)through many cities in honoring(B)through many cities to honor(C)in many cities; it was to honor(D)by many cities to honor(E)by many cities in honoring9. During the 1980’s, the income gap between the richest and the poorest Americans widenedsignificantly, while continuing to expand in the 1990’s.(A)significantly, while continuing to expand(B)significantly, and it continued to expand(C)significantly with continuing expansion(D)significantly, it continued expanding(E)significantly, continuing expanding14. High school graduates usually do not end up earning as much income as college graduatesdo, this being why so many high school students go on to pursue college degrees.(A)do, this being why so many high school students(B)do, this is why so many high school students(C)do; this fact explains why so many high school students(D)do; this fact explaining the reason for why so many high school students(E)do, explaining why so many high school studentsOG Test 9Section 33. Legend has it that medieval Chinese warriors using manned kites to survey enemy troopsanticipating modern aerial surveillance.(A)using manned kites to survey enemy troops anticipating(B)using manned kites to survey enemy troops and anticipate(C)using manned kites and surveying enemy troops, they anticipated(D)used manned kites to survey enemy troops, a technique anticipating(E)used manned kites surveying enemy troops, so anticipating techniques of5.The programmers always talked of having too much to do, but in truth they had a lesseramount of work to do than their colleagues.(A)they had a lesser amount of work to do than(B)their work was the least among(C)they were having less work to do than(D)the amount of work they had to do was the least of(E)they had less work to do than7. Unlike American architects who preceded him, Frank Lloyd Wright did not draw on classicalor European architecture for inspiration.(A)Unlike American architects who preceded him, Frank Lloyd Wright(B)Unlike the inspiration of American architects who preceded him, Frank Lloyd Wright (C)Frank Lloyd Wright’s architecture, unlike American architects who preceded him,(D)Different from the American architects who preceded him, Frank Lloyd Wright’s designs (E)Frank Lloyd Wright’s inspirations, different from American architects who preceded him,10. In Germany, foresters discovered that trees killed by acid rain had begun to die four yearsearlier, even though the trees had shown no signs of disease then.(A)earlier, even though the trees had shown no signs of disease then(B)earlier, but not showing any signs of disease then(C)earlier, no outward signs of disease had been shown then in the trees, however(D)earlier, without any signs of disease shown then(E)earlier, not then having shown any signs of disease, however。
SATOG2阅读填空解析(二)-智课教育出国考试
智 课 网 S A T 备 考 资 料SATOG2阅读填空解析(二)-智课教育出国考试考生必须对SAT阅读考试的种种特点和具体的题型的充分了解,这样才能为他们获取SAT的高分打下了坚实的基础。
下面来看下SATOG2阅读填空解析。
SAT阅读考试是中国考生相对比较头痛的一个考试项目,所以对于想要拿到SAT阅读高分的考生来说,多掌握一些词汇是十分必要的。
下面就为大家介绍一下SATOG2阅读填空解析,供大家参考。
Section 7P4751. Geoffrey's corrupt dealings earned him such disgrace that any possibility of his being reelected to the city council was completely _________.(A) ensured adj. 确保(B) approved adj. 被认可的(C) belittled adj. 轻视(D) eliminated adj. 消除,除去, 完全没有了的(E) defended adj. 保卫,防御答案:D解析:递进关系翻译:Geoffrey的腐败行为使他名誉扫地,他再次入选市委员会的概率完全的_________。
2. Although the editors were reputed to be very _________, the uneven quality of the material they put into the anthology suggests they were too _________.(A) amateurish adj. 外行的...professional adj. 专业的(B) lax adj. 不严格的...harsh adj. 严厉的,丑陋的(C) selective adj. 选择的,不普遍的...inclusive adj. 范围广泛的(D) judgmental adj. 审判的...discriminating adj. 有鉴别能力的(E) sensitive adj. 敏感的...insightful adj. 富有洞察力的答案:C解析:转折对比关系翻译:虽然编辑者被认为是非常_________,他们选进诗文选集里的不同特征的素材暗示他们太_________。
SAT OG阅读文本TEST4
3The passages below are followed by questions based on their content; questions following a pair of related passages may also be based on the relationship between the paired passages. Answer the questions on the basis of what is stated or implied in the passages and in any introductory material that may be provided.Questions 6-9 are based on the following passages. Passage 1The eighteenth-century botanist Carolus Linnaeus'enormous and essential contribution to natural historywas to devise a system of classification whereby anyplant or animal could be identified and slotted into5 an overall plan. Yet Linnaeus himself would probably have been the first to admit that classification is onlya tool. and not the ultimate purpose. of biological inquiry. Unfortunately, this truth was not apparentto his immediate successors, who for the next hundred10 years were to concern themselves almost exclusivelywith classification.Passage 2I am a heretic about Linnaeus. Ido not dispute thevalue of the tool he gave natural science, but I am waryabout the change it has effected on humans' relationship 15 to the world. From Linnaeus on. much of science has been devoted to sorting masses into individual entitiesand arranging the entities neatly. The cost of having so successfully itemized and pigeonholed nature is to limitcertain possibilities of seeing and apprehending. For20 example. the- modem human thinks that he or she can best understand a tree (or a species of tree) by examininga single tree. But trees are not intended to grow in isolation. They are social creatures. and their society in tum supports other species of plants. insects. birds. mammals. and micro- 25 organisms. all of which make up the whole experience of the woods. '6. Compared to the author of Passage 2. the authorof Passage l regards Linnaeus with more(A) cynicism(B) bafflement(C) appreciation(D) nostalgia(E) resentment7 Unlike the author of Passage l, the author of Passage 2 makes use of(A) scientific data (B) literary allusion(C) historical research(D) personal voice(E) direct citation8 Both passages emphasize which of the following aspects of Linnaeus' work?(A) The extent to which it contributed to naturalscience(B) The way in which it limits present-day science(C) 'Die degree to which it revived interest in biology(D) 'l`he decisiveness with which it settled scientific disputes(E) The kinds of scientific discoveries on whichit built `9 The author of Passage 1 would most likely respondto the opening of Passage 2 (lines 12- I7) by arguingthat the author of Passage 2 has(A) demonstrated that Linnaeus should be betterknown as a scientist than he currently is(B) minimized the achievements of those scientistswho built on Linnaeus' work(C) refused to appreciate the importance of proper classification to scientific progress(D) failed to distinguish the ideas of Linnaeus fromthose of his followers(E) misunderstood Linnaeus` primary contributionto natural historyQuestions 10-15 are based on the following passage.The following is an excerpt from a translation of a novel written in Spanish by an author from Colombia. In a fanciful manner. the novelist portrays the townspeopleof an isolated village.Dazzled by so many and such marvelous inventions, the people of Macondo did not know where their amazement began. They stayed up all night looking at the pale electric bulbs fed by the electric plant that Aureliano Triste had5 brought back when the train made its second trip, and ittook time and effort for them to grow accustomed to itsobsessive noise.They became indignant over the living images that the prosperous merchant Bnino Crespi projected on the screen 10 in the theater with the lion-head ticket windows, for the character who had died and was buried in one film, and for whose misfortune tears of affliction had been shed, would reappear alive and transformed into an Arab sheik in the next one. The audience. who paid two cents apiece to share 15 the difficulties of the actors, would not tolerate such an outlandish fraud and they broke up the seats. The mayor,at the urging of Bruno Crespi, explained in a proclamation that the cinema was a machine of illusions that did not merit the emotional outbursts of the audience. With that20 discouraging explanation many felt that they had been the victims of some new trickery and they decided not to return to the movies. considering that they already had too many troubles of their own to weep over the acted-out misfor- tunes of imaginary beings.25 Something similar happened with cylinder phonographsbrought from France and intended as a substitute for theantiquated hand organs used by the band of musicians. Fora time the phonograph records had serious effects on thelivelihood of the musicians. At first curiosity increased the 30 business on the street where they were sold and there waseven word of respectable persons who disguised them-selves as workers in order to observe the novelty of thephonograph at firsthand. but from so much and such close observation they soon reached the conclusion that it was 35 not an enchanted mill as everyone had thought and as somehad said, but a mechanical trick that could not be compared with something so moving, so human, and so full of every- day truth as a band of musicians. It was such a serious disappointment that when phonographs became so popular 40 that there was one in every house they were not considered objects for amusement for adults but as something good for children to take apart.On the other hand, when someone from the town had the opportunity to test the crude reality of the telephone45 installed in the railroad station, which was thought to bea rudimentary version of the phonograph because of itscrank. even the most incredulous were upset. It was as ifGod had decided to put to the test every capacity for sur-prise :ind was keeping the inhabitants of Macondo in a50 permanent alternation between excitement and disappoint-ment. doubt and revelation, to such an extreme that no oneknew for certain where t.he limits of reality lay.10 The word "obsessive" (line 7) most nearly means(A) enthusiastic(B) persistent(C) obvious(D) infatuated(E) hardworking11 The "fraud" (line 16) that upset the citizens of Macondowas related to the(A) excessive charge for admission(B) outlandish adventures of the characters on the screen(C) fact that the events depicted on the screen did not actuallyoccur(D) types of difficulties the actors faced(E) implausible plots of the stories that were told12 The citizens lost interest in their phonographs(A) the machines lacked the heart and soul of true musicians(B) few people were able to operate them(C) the machines were too difficult to observe firsthand(D) many musicians lost their jobs because of(E) the children were breaking them faster than were made13 The citizens of Macondo were distressed by the of thetelephone because they(A) did not know where it had come from(B) had expected a more socially beneficial invention(C) could envision the changes it would bring to daily village life(D) no longer felt able to make the usual assumptions about their world(E) were fearful that it would have serious effects on their continued employment14 The aspect of the new inventions that most disappointed the citizens was that these inventions(A) were not all fashioned with a crank(B) did not have any real educational value(C) were not at all what they seemed to be(D) were meant purely for entertainment(E) were so intricate they were difficult to operate15 The major purpose of the passage is to(A) illustrate the influence the distinguished residents of Macondo had on the other citizens(B) describe the new scientific inventions that were introduced to Macondo(C) depict a diverse crowd reacting in unison to amagical performance(D) describe the people's responses to the influx oftechnical advances(E) delineate old-fashioned ideas about the virtue ofnature over technologyQuestions 16-24 are based on the following passage.This passage is by a choreographer who worked with the influential dancer and choreographer Martha Graham (1894-1991). It focuses on the use of space and gesture in dance.I am not an adept aesthetician, and I could not presume to an alyze Nlartha’s sense of design or approach toward design. But I believe she dealt with the elements of line and direction with the instincts of a mathematician or physicist.5 adding to each their emotional relations. For example. astraight line rarely, if ever, occurs in nature, but it doesoccur in art, and it is used in art with various tellingeffects. Direction works similar magic. An approachingbody produces one kind of emotional line, a receding or 10 departing body another; the meeting of two forces produces visual. kinesthetic, and emotional effects. with a world of suggestibility around them like a penumbra that evokes many ideas and emotions whenever these forms are manip- ulated. Basic human gesture: assume, therefore. an almost 15 mystic power. The simple maneuver of turning the face away. for example, removes personality, relationship. Not only mat, it seems to alter the relation of the individual to present time and present place, to make here-and-now other-where and other-time. It also shifts the particular20 personality to the general and the symbolic. This is the power of the human face and the human regard. and the meeting of the eyes is probably as magic a connection as can be made on this earth. equal to any amount of electrical shock or charge. It represents the heart of dynamism. life 25 itself. The loss of that regard reduces all connections to nothingness and void."Turning one`s back" has become a common figure of speech. It means withholding approval. disclaiming. negat- ing: and, in fact. in common conduct the physical turning 30 of the back is equated with absolute negation and insult.No back is turned on a royal personage or :r figure of high respect. This is linked with the loss of visual contact and regard. One cuts dead by not meeting the eyes.We know much about emotional symbols. Those used35 by the medieval and Renaissance painters were understoodby the scholars and artists of the time-but. more wonder-ful. they mean to us today spontaneously just what tl1ey meant then; they seem to be permanent. We dream. Jung`tells us. in terms and symbols of classic mythology. And40 since. according to Jung, all people share a “collectiveunconscious." people from disparate traditions nonethelessdream in the same terms. ls it not also likely. then, thatcertain space relations, rhythms. and stresses have psy-chological significance. that some of these pattems are45 universal and the key to emotional response. that theirdeviations and modifications can be meaningful to artistsin terms of their own life experiences and that these over-tones are grasped by spectators without conscious arzalysis?These matters are basic to our well-being as land and air50 animals. As plants will tum to sunlight or rocks or moisture according to their nature, so we bend toward or escapefrom spatial arrangements according to our emotional needs. Look around any restaurant and see how few people will sit at a center table unless the sides are filled up. Yet55 monarchs of old always dined dead center and many Limqin public.The individual as a personality. then, has a particular :code in space and rhythm, evolved from his or her life Qhistory and from the history of the human race. It is just60 the manipulation of these suggestions through time-space that is the material of choreography. ‘*A Swiss psychologist (1875-1961)16. The first two sentences (lines l-5) are characterized, respectively, byA disclaimer and assertionB invocation and definitionC apology and confessionD authority and hypothesisE rebuttal and analysis17. In lines 5-6. the statement "a straight line rarely, if ever. occurs in nature” emphasizes the author’s recognition of the(A) choreographer’s need for spectacular effects(B) choreographer’s use of mathematical forms(C) choreographer’s estrangement from nature(D) impossibility of performing certain choreographer motions(E) universality of geometrical forms18. By saying that the meeting of two forces produces effectsthat have "a world of suggestibility around them" (lines l l-12) the author means that the physical event(A) provokes unwarranted suspicions(B) reveals the motives of the artist(C) acts on the gullibility of the audience(D) lulls the audience into complacent acceptance(E) evokes a vast number of associations19. The authors main point about "human gestures"(line 14) is that they(A) are not subject to an individuals control( B ) are difficult to analyze without scientific terminology(C) provoke different responses in people(D) carry powerful. universally understood messages(E) evolve with changes in cultural hierarchy20 The author mentions "the meeting of the eyes" '(lines 2|-22) to suggest the(A)effect that tank or status has on gestural meaning(B)difficulty of controlling emotional symbols(C)degree to which body language is not a function of personality(D)extent of the power of individual human contact(E)nature of artistically pleasing events21 The author suggests which of the following aboutthe work of "medieval and Renaissance painters" (line 35) ?(A) It was influenced by its royal patronage.( B) It was conceived more spontaneously than is modem art.(C) It should be cherished for its unique symbolism.(D) It contains symbols that are immediately accessible to contemporary viewers.(E) It is an unsophisticated version of symbolism developed later by choreographers.22 As used in line 43. "stresses" most nearly means(A) emphases(B) loads(C) anxieties(D) influences(E) sounds23 As used in line 48. "grasped" most nearly means(A)adhered to(B) seized on(C) controlled(D) held(E) understood 24 The author suggests that people in a restaurant (lines 53-54) are expressing their emotional need for(A) unhindered interaction(B) relative privacy(C) respect from strangers(D) approval from others(E) reclusive isolation7The passages below are followed by questions based on their content; questions following a pair of related passages may alsobe based on the relationship between the paired passages. Answer the questions on the basis of what is stared or implied in the passages and in any introductory material that may be provided.Questions 9~l0 are based on the following passage.When the tide was in and the water rose up to withina foot of the lawn. we children boasted that we could fishout of our bedroom windows. This was not quite true. But it was true that, from our front lawn, the house was full of 5 waves. When the tide was up and the sun was shining. the white front of the house was in movement with reflected waves. The tall windows became so solid in color and form, gold and blue. that the house seemed to be hill of Sea; until, of course. one turned round and saw the teal10 sea. so miraculously real that it startled.9. What does the narrator mean by the comment “the house was full of waves” (lines 4-5)(A) The house ws decorated with a nautical theme.(B) The children enjoyed drawing pictures of the sea.(C) The house mirrored the movements of the sea.(D) 'The basement of the house sometimes killed with water.(E) The house appeared as if it had been damaged by the sea.10. In context. the tone of lines 5-10 (“When . . . startled") is best described as one of(A) awe and fear(B) mischief and curiosity(C) sadness and confusion(D) wonder and delight(E) uncertainty and impatienceQuestions 11-12 are based on the following passage.Recently excavated artifacts from Pakistan have inspireda reevaluation of one of the great early urban cultures--theenigmatic Indus Valley civilization. one of the four greatearly Old World state-cultures, along with Mesopotamia,5 Egypt, and China's Yellow River civilization Much lessis known about the Indus civilization than these other states because linguists have yet to decipher the Harappan script found on recovered objects. Attempting to understand these vanished people and their social structures, my colleagues 10 and I have drawn clues from the miscellaneous objects we uncover and sites we excavate. In this effort, the Harappan writings have not been totally useless; we have gleanedinsights by examining the context of the writing's use.11. A major assumption of the passage is that(A) the spot within an excavated site where an object is found is a clue to its social significance(B) it is a great help in understanding a civilization tobe able to decode its language'(C) there are similarities among the social structures of ancient urban civilizations(D) an effective archaeologist should learn the language of the civilization being studied(E) ancient languages are all very difficult to decipher12. 'The author’s tone in the final sentence is best described as(A) frustrated(B) resigned(C) ambivalent(D) somewhat encouraged(E) unshakably confidentQuestions 13-24 are based on the following passages.These were two passages. written in the 1990’s. address the ways in which environmental consent: have been made public. Passage 1There is nothing wrong with attempting to make theoften difficult and complex Endings of science availableto a wider audience. but environmental popularizers often present a one-sided picture and hide important scientific5 disagreements on issues relevant to environmental quality.The zeal to draw firm conclusions from the results of sci-entific research frequently prompts speculative mattersto be left out or presented with greater authority than theydeserve. The partisanship implicit in these failures is most10 often excused by the originality of the author’s perspectiveon the subject or a passionate commitment to do good.H ow could one regret the “minor” confusions that mightarise from such noble impuises?But using one-sided and incomplete accounts of the state15 of scientific knowledge has led to projections, predictions, and warnings that. not surprisingly, have been falsified byevents. No one knows what the future holds. But reports that Lake Erie and the oceans would be dead by now were surely greatly exaggerated. The United States is wracked 20 neither by food riots nor a great epidemic of pesticide- induced cancers. Birds continue to sing in the mornings, and they do not have to face the rigors of either an ice age caused by humans or a global warming caused by the heat of increased energy production and consumption. With25 what confidence should we look upon the projected honors of global warming. rain forest destruction, or toxic waste, given the record of the past?This failure of prophecy may be an intellectual weakness. yet prophecy continues because it provides the popularizers 30 with a profound rhetorical strength: it releases the power of fear. 'lite central role of this sentiment in political rhetoric has long been understood. Arousing fear. though. is not always easy. Even as far back as Aristotle. it was observed that we fear things less the more distant they are. Hence35 when Churchill sought to rouse the British. he brought the Germans to the beaches. landing grounds, fields. streets. and hills of "our island.“ So. too. to arouse fears the popu- larizers have to present pict1.tres of imminent calamities that could befall their relatively comfortable and well-off 40 readers. Environmental disasters like endemic waterbome disease due to inadequate sewage treatment in faraway nations do not Et this category. The prospect of my getting skin cancer due to ozone depletion does. Without such immediacy. one could only arouse a sentiment like com-45 passion. which is not as strong as fear.Passage 2Few ideas are more deeply entrenched in our political culture than that of impending ecological doom.in l962., when Rachel Carson wamed readers that polluting was a threat to all life on the planet, pessimistic appraisal;50 of the health of the environment have been issued with increasing urgency. And yet. thanks in large part to her Q warnings. a powerful political movement was born and a series of landmark environmental bills became law: the fs; Clean Air Act (1970). the Clean Water Act (l972), and55 the Endangered Species Act (1973). These laws and their ` equivalents i.n Western Europe, along with a vast array of private efforts, have been a stunning success. ln both the United States and Europe, environmental trends are, forth; most part, positive; and environmental regulations, far from 60 being burdensome and expensive, have proved to be strik--ingly effective, have cost less than was anticipated, andhave made the economies of the countries that havethem into effect stronger. not weaker.Nevertheless, the vocabulary of environmentalism has65 continued to be dominated by images of futility. crisis, anddecline. In 1988. Thomas Berry, an essayist popular amongec ologists. wrote that “the planet canno t long endure present modes of human exploitation.” ln 1990, Gaylord Ne lsonthe former senator from Wisconsin who was a prime mover 70 behind the first Earth Day in l970, said that environmental problems “are a greater threat to Earth's life-sustaining systems than a nuclear war." And in l993 Vice PresidentAl Gore said that the planet now was sufferi ng ‘grave and perhaps irreparable damage." But. at least insofar as the75 Western world is concerned. this line of thought is an anachronism. rendered obsolete by its own success. Norare environmentalists the only people reluctant to acknowl- edge the good news: advocates at both ends of the political; spectrum. each side for its own reasons, seem to have80 tacitly agreed to play it down. The Left is afraid of the environmental good news because it undercuts stylish pessimism: the Right is afraid of the good news becauseit shows that government regulations might occasionally amount to something other than wickedness incarnate and85 actually produce benefits at an affordable costThis is a bad bargain-for liberals especially. Their philosophy is under siege on many fronts-crime welfare? medical care, and education. among others. So why not trumpet the astonishing, and continuing, record of success90 in environmental protection?13. l.n line 14, “state" most nearly means(A) rank(B) excitement(C) territory(D) government(E) condition14.The author of Passage l indicates that "food riots" (line 20) and "pesticide-induced cancers" (lines 20~21) are(A)problems the nation will ultimately encounter(B)problems facing underdeveloped areas of the world(C)among the predictions of environmental popularizers(D)among the consequences of global warming(E)potential results of the pollution of lakes and oceans15 The term "rigors" in line 22 refers to(A)the efforts needed for environmental cleanup(B)stringent regulations put in place since 1970(C)moralistic attitudes of many environmental popularizers(D)projected consequences of environmental decline(E)ability of nature to recover from environmental abuse16 The author of Passage l uses the example in lines 42-43 ("The prospect. . . does") to(A)describe a personal experience(B)imply that the subject should not be frightening(C)elicit sympathy from the reader(D)demonstrate a psychological fact(E)emphasize the prevalence of a crisis17 The first paragraph of Passage 2(lines 46-63) presents(A)an elaborate speculation(B)a historical summary(C)a list of sources(D)an Introductory aside(E)a scientific theory18 ln Passage I. the phrase "rendered . . success" (line76)indicates that(A)the desires of environmentalists have changed over the years(B)the success of the environmental movement has frightened conservative politicians(C)the accomplishments of the environmental movement have made its public pronouncements irrelevant(D)environmentalists often appear old-fashioned in a world primarily concerned with technology(E)environmentalism plays on the political concerns of both liberals and conservatives19 The Phrase "wickedness incarnate" (line84)is used to(A)cast aspersions on bureaucratic ineptitude(B) parody the language used by people with certain political leanings(C) convey humorously a deep longing of the author(D) rail against blatant polluters of the environment(E)suggest the quasi-religious underpinnings of environmentalism20 The attitudes toward environmentalism of the authors of Passage l and Passage 2. respectively. are(A)outrage and resentful disappointment(B)skepticism and qualified admiration(C) indifference and urgent concern(D) alarm and grudging acceptance (E) open-mindedness and staunch advocacy21 What would the author or Passage I most likely say about the sort of reports mentioned in lines l7~l9 of Passage l ?(A) They were unethical attempts to manipulate public opinion(B) They reflected the scientific uncertainty of their era.(C) They seem quite naive in retrospect.(D) They served a purpose in their time.(E) They are needed today more than ever.22 The author of Passage l would most likely argue that the "line oftl1ought" (line 75) illustrated in Passage 2 was(A) once original hut is now trite(B) once wholly based on science but is now driven by politics(C) in no way meant to be taken literally(D) of no significance to faraway nations(E) of dubious validity from the beginning23 How would the author of Passage l be most likely to answer the question posed at the end of Passage 2(A)Because good news is less of a stimulus to action than are dire warnings(B)Because environmentalists fear alienating either the Left or the Right(C)Because environmentalists themselves are divided about whether their task has been accomplished(D)Because boasting is still considered in appropriate by the liberal elite(E)Because laypersons lack the training to evaluate the environmental record24 The authors of Passage l and Passage 2 agree that(A)the state of the environment continues to worsen(B)the environmental movement lacks political influence(C)most of the information citizens receive about the environment is overly technical(D)spokespeople for the environmental movement art not sufficiently knowledgeable(E)the environmental movement employs exaggerated rhetoric9The passage below is followed by questions based on its content. Answer the questions on the basis of what is stated or implied in the passage and in any introductory material that may be provided.Questions 7-19 are based on the following passage. Frederick Douglass (1817-1895), who escaped from slavery, became an author and publisher and was internationally known for his instrumental role in the abolitionist movement.In spite of the ridicule that various newspapers aimed at the women’s movement, Frederick Douglass continued to lend it his active support. Indeed, few women’s rights con- ventions were held during the l850’s at which Douglas s5 was not a featured speaker and whose proceedings were notfully reported in his paper. Invariably. the notice would be accompanied by an editorial comment hailing the meeting and expressing the editorial hope that it “will have a power- fu l effect on the public’s mind." In 1853,`when Douglass 10 was considering changing the name of his newspaper, he rejected the proposed title, The Brotherhood, because it “implied the exclusion of the Sisterhood." He called itFrederick Douglass' Paper, and underneath the title werethe words "All Rights For All!" '15 Because women were not permitted to speak at mass meetings of state temperance associations,| women in New York formed the Woman's State Temperance Society, with Elizabeth Cady Stanton as president. Douglass supported the society but took issue with the move led by secretary20 Amelia Bloomer to limit to women the right to hold its offices. He aligned himself with Stanton and Susan B. Anthony in opposing this as a violat ion of “the principleof human equality"-a violation, in short. of men’s rights. Douglass felt that by excluding men from office the society 25 would lose supporters in the battle against those in the temperance movement who wished to deny women equal rights. l-low. he asked. could women effectively contendfor equality in the movement when they denied it to men?ln lune l853. the society accepted the logic of this position 30and admitted men to office.Douglass leamed much from women with whom he associated at the national and state women`s rights conven- tions. At one time, he had entertained serious doubts about wives being given the tight to share equally with their 35 husbands the disposition of property. since “the husband labors hard" while the wife might not be earning money. But his discussions with pioneers of the women’s rights movement convinced him that even though wives were not paid for their domestic labors, their work was as important 40 to the family as that of their husbands. Once convinced, he acted. He wrote the call for the 1853 convention in Rochester, New York. which demanded not only that women be paid equally with men for their work. but also that women. including married women. have equal rights 45 with men in the ownership and disposition of property. ln his newspaper that year. Douglass urged state legislation calling for passage of a law requiring equality in "the holding. and division of real and personal property.”On one issue, however,Douglass refused to budge.50 He was critical of women’s rights leaders who addressed audiences from \which Black people were barred. His particular target was Lucy Stone. Douglass often praised this abolitionist and veteran lighter for equal tights for women. but he criticized her for not having canceled a _ 55 lecture in l853 at Philadelphi a’s Music Hall when she discovered that Black people would be excluded. Later.he was more severe when he learned that she had invited Senator Stephen A. Douglas of Illinois, one of the archi- tects of the infamous Fugitive Slave Act of 1850.3 to join 60 the women who were to meet in Chicago in i859 to pub- licize the women’s rights cause. Frederick Douglass bluntly accused Stone of willingness to advance women’s rights on the back of “the defenceless slave woman" who "has alsoto bear the ten thousand wrongs of slavery in addition to 65 the common wrongs of woman."Douglass' disputes with some of the women’s rights leaders went beyond the question of their appearance before segregated audiences. Women like Stanton and Anthony were close to abolitionist William Lloyd Garrison.70 When Douglass split with Garrison over the latter’s reli-ance on words and “moral suasion” as the maj or route to abolition. as well as over Garrison`s opposition to anti-slavery political action, some women’s tights leaders grew。
SAT OG阅读文本TEST3
3Each passage below is followed by questions based on its content. Answer the questions on the basis of what is stated or implied in each passage and in any introductory material that may be provided.Questions 6-7 are based on the following passage.The critic Edmund Wilson was not a self-consciousletter writer or one who tied to sustain studied manner-isms. Nor did he resort to artif ice or entangle himself incircumlocutions. The young, middle-aged, and old Wilson5 speaks directly through his letters, which are inf ormal f orthe most part and which undisguisedly ref lect his changingmoods. On occasion~in response, perhaps, to the miseryof a f ri end or a public outrage or a personal challenge-hecan become eloquent, even passionate, but that is not his10 prevailing tone.6. Based on the information in the passage, Wilson’s letters can best be described as(A) cynical(B) spontaneous(C) critical(D) preachy(E) witty …7. The reference to the "young, middle-aged, andold Wilson" (line 4) serves to suggest the(A) multif aceted nature of Wilson‟s literarypersona(B) maturity Wilson displayed even as a youth(C) effe ct aging had on Wilson‟s temperament(D) longevity of Wilson‟s literary career(E) consistency of Wilson‟s letter-writing styleQuestions 8-9 are based on the following passage.The belief that it is harmf ul to the Black communityf or authors to explore the humanity of our leaders canhave troubling ef fects. At the least, it promotes the beliefthat our heroes have to be perf ect to be usef ul. At worst,5 it censors our f ull investigation of Black lif e. If our paint-ings of that lif e are stock and cramped, their colors draband predictable, the representations of our culture are likelyto be untrue. They will not capture the breadth andcomplexity of Black identity.8. The passage implies that Black leaders havesometimes been portrayed as being(A) overly sentimental (B) deeply complex '(C) above reproach(D) without regret(E) beyond understanding t9. In context, the “paintings” (lines 5-6) are best understood as a reference to(A) realistic sculptures(B) historical biographies(C) whimsical novels(D) political cartoons(E) colorf ul theoremsQuestions 10-18 are based on the following passage.The following passage was written by a physicist in 1986.When astronomers point their telescopes to the nearest galaxy, Andromeda, they see it as it was two million years ago. That‟s about the time Australopithecus* was baskingin the Af ri can sun. This little bit of time travel is possible5 because light takes two million years to make the trip f romthere to here. Too bad we couldn‟t tum- things around and observe Earth f rom some cozy planet in Andromeda.But looking at light f rom distant objects isn‟t real timetravel, the in-the-f lesh participation in past and f uture f ound10 in literature. Ever since I‟ve been old enough to read s ciencef iction, I‟ve dreamed of time traveling. The possibilitiesare staggering. Y ou could take medicine back to f ourteenth- century Europe and stop the spread of plague, or you could travel to the twenty-third century, where people take their15 annual holidays in space stations.Being a scientist myself, I know that time travel is quite unlikely according to the laws of physics. For onething, there would be a causality violation. If you couldtravel backward in time, you could alter a chain of events20with the knowledge of how they would have turned out.Cause would no longer always precede effect. For exam-ple, you could prevent your parents f rom ever meeting.Contemplating the consequences of that will give you aheadache. and science f i ction writers f or decades have25 delighted in the paradoxes that can arise from travelingthrough time.Physicists are, of course, horrif i ed at the thought ofcausality violation. Dif ferential equations f or the waythings should behave under a given set of f orces and30 initial conditions would no longer be valid, since whathappens in one instant would not necessarily determinewhat happens in the next. Physicists do rely on a determin- istic universe in which to operate, and time travel would almost certainly put them and most other scientists35 permanently out of work.Still, I dream of time travel. There is something very personal about time. When the f irst mechanical clockswere invented, marking off time in crisp, regular inter-vals. it must have surprised people to discover that time40 f lowed outside their own mental and physiological pro-cesses. Body time f lows at its own variable rate, oblivious to the most precise clocks in the laboratory. In f act, thehuman body contains its own exquisite timepieces, all with their separate rhythms. There are the alpha waves in the45 brain; another clock is the heart. And all the while tick themysterious, ruthless clocks that regulate aging.Recently, I f ound my great-grandfather‟s favorite pipe.Pa a Joe as he was called died more than seventy yearsago, long bef ore I was born. There are f ew surviving photo- 50graphs or other memorabilia of Papa Joe. But I do have his pipe, which had been tucked away in a drawer somewheref or years and was in good condition when I f ound it. l rana pipe cleaner through it, f illed it with some tobacco I hadon hand, and settled down .to read and smoke. Af ter a cou- 55 ple of minutes, the most wonderf ul and f oreign blend ofsmells began waf ting f rom the pipe. All the dif ferent occa- sions when Papa Joe had lit his pipe, all the diff erent places he had been that I will never know-all had been lockedup in that pipe and now poured out into the room. I was60 vaguely aware that something had got delightf ully twistedin time f or a moment, skipped upward on the page. There is a ki nd of time travel to be had, if you don‟t insist on how it happens.* An extinct humanlike primate10. The aut hor mentions Australopithecus in line 3in order to(A) note an evolutionary progression in the physicalworld(B) dramatize how different Earth was two million years ago(C) commend the superior work of astronomers inisolating a moment early in time(D) establish a link between the length of time thatAf rica has been inhabited and the discovery ofthe Andromeda gal axy(E) emphasize the rel atively long period of humanlif e compared to the age of the universe11. The statement in lines 6-7 (“Too bad . _ . Andromeda”) suggests that .(A) scientists would like to observe events that occurred on Earth in the distant past(B) there m ay be planets in Andromeda that are reachable through space travel(C) the study of Andromeda would offer interesting comparisons to planet Earth(D) a planet in Andromeda will be a likely observation point f or Earth in the f uture _(E) Androm eda is much_older than Earth12. The aut hor mentions “plague” (line 13) and“space stations” (line 15) primarily to(A) give an example of the themes of novels about time travel(B) suggest contrasting views of the f uture(C) scoff at the scientif ic consequences of time travel(D) give examples of the subjects that scientists are interested in(E) suggest why time travel is such a f ascinating topic13 The author introduces the third paragraph with t he' words “Being a scientist" in order to(A) explain an intense personal interest in the topic(B) lend an air of authority to the discussion of time travel(C) suggest why certain f orms of literature are so appealing(D) provoke those who def end science f iction(E) help illustrate the term “causality violation”14 In discussing causality violations (lines 16-35), theauthor addresses concerns about all of the following EXCEPT(A) anticipatory knowledge of events(B)the belief in a deterministic universe(C)the mechanics of space travel(D)cause-and-effect relationships(E)di ff erential equations based on known f orces15 Which of the following, if true, would undermine the validity of t he author’s assumption abou t the impact of mechanical clocks (“When the first _ . _ the laboratory”)in lines 37-42 ?(A) People were oblivious to time on a physical levelbef ore clocks were invented.(B) People have always perceived time as composedof discrete, unif orm intervals.(C) Concern about time was unnecessary until clockswere invented.(D) Mental and physiological processes are verypredictable.(E) Body time does not move at a constant rate.16 The author mentions the brain and the heart(lines 44-45) in order to(A) demonstrate the rhythmical qualities of timepieces(B)explain the historical signif icance of mechanicalclocks(C) emphasize how the two organs interact to regulateinternal rhythms(D)illustrate the body‟s diff erent inte rnal clocks(E)demysti f y the precision of organic processes17 The author uses the word “rut hless” (line 46) tosuggest that -(A) people are bewildered by the prospect of aging(B) the human body has mysterious capacities(C) some people age more rapidly than others do(D) people‟s sense of time changes as they age(E) the process of aging is relentless18 The author mentions that “something _ . . skipped upward on the page” (lines 60-61) to suggest that(A) he reread a portion of the page(B) his vision was aff ected by the smoke(C) he traveled back in time in his imagination(D) his reading reminded him of Papa Joe(E) he believes that reading is the best way to recreatethe pastQuestions 19-24 are based on the following passage.The following passage is an excerpt from a book about twentieth-century developments in art. The author refershere to the modern art that emerged shortly after the turnof the century. Many people found this art shocking.If the new art is not accessible to everyone, which certainly seems to be the case, this implies that its impulsesare not of a generically human kind. It is an art not f orpeople in general but f or a special class who may not be 5 better but who are evidently diff erent.Bef ore we go f urther, one point must be clarif ied. Whatis it that the majority of people call aesthetic pleasure? What happens in their minds when they “like” a work ofart; f or example, a play? The answer is easy. They like a10 play when they become interested in the human destiniesthat are represented, when the love and hatred, the joys and sorrows of the dramatic personages so move them that they participate it all as though it were happening in real life.And they call a work “good” if it succeeds in creating the15 illusion necessary to make the imaginary personages appearlike living persons. In poetry the majority of people seekthe passion and pain of the human being behind the poet.Paintings attract them if they f ind in them f igures of men or women it would be interesting to meet.20 It thus appears that to the majority of people aestheticpleasure means a state of mind that is essentiallyindistinguishable f rom their ordinary behavior. It dif fersmerely in accidental qualities, being perhaps less utilitarian, more intense, and f ree f rom painf ul consequences. But the 25 object toward which their attention and, consequently, alltheir other mental activities are directed is the same as indaily lif e: people and passions. When f orced to considerartistic f orms proper-f or example, in some surrealistic orabstract art-most people will only tolerate them …if they do30 not interf ere with their perception of human f om1s andf ates. As soon as purely aesthetic elements predominateand the story of John and Susie grows elusive, most peoplef eel out of their depth and are at a loss as to what to makeof the scene, the book, or the painting. A work of art35 vanishes f rom sight f or a beholder who seeks in that workof art nothing but the moving f ate of John and Susie or Tristan and Isolde.* Unaccustomed to behaving in anymode except the practical one in which f eelings are aroused and emotional involvement ensues, most people are unsure 40 how to respond to a work that does not invite sentimentalintervention. *Now this is a point that has to be made perf ectly clear.Neither grieving nor rejoicing at such huma.n destinies asthose presented by a work of an begins to def ine tme45 artistic pleasure; indeed, preoccupation with the humancontent of the work is in principle incompatible withaesthetic enjoyment proper.* Tristan and Isolde were star-crossed lovers in a medievalromance.19 The passage is primarily concerned with the(A) lives artists lead as opposed to the ones they imagine(B)emotional impact of a painting‟s subject matter(C)nature of the pleasure that most people f ind in awork of art(D) wide variety of responses that audiences have to different works of art(E) contrast between the f ormal elements of the newart and those of the old20 As used in line 18, “figures” most nearly means(A) crude images(B) abstractions(C) representations(D) numbers(E) f amous persons21 It is most likely that “the story of John and Susie”(line 32) refers to(A) a f ictional work that the author will proceed tocritique(B) a typical narrative of interpersonal relationships(C) an account of an affair in the f orm of a mystery(D) a legendary couple that has fascinated artiststhrough the ages(E) a cryptic chronicl e of renowned historicalpersonages '22The author suggests that the majority of people resist modern art because they(A) consider modem artists to be elitist(B) are too inf luenced by critics to view the art on itsown merits(C) are annoyed by its social message(D) f ind in it little of human interest to engage them(E) f ind it too diff icult to guess at the artist‟s sourceof inspiration23 The author’s attitude toward the majority of people can best be described as(A)genuinely puzzled(B)aggressively hostile(C)solemnly respect f ul(D)generally indiff erent(E)condescendingly tolerant24 The author’s assumption in the final paragraph(lines 42-47) is that (A) aesthetic pleasure is a response to the purely artistic elements in a work of art(B) aesthetic enjoyment of a work of art must f ocus on the artist‟s intentions as much as on the artist‟s actual accomplishments '(C) responses to a work of art vary and cannot be easily def ined(D) the evocation of emotional responses by a traditional work of art depends on the moral conventions of the artist‟s society (E) the majority of peopl e trying to interpret a work of art will concentrate on the artistic technique7The passages below are f ollowed by questions based on their content; questions f ollowing a pair of related passages may alsobe based on the relationship between the paired passages. Answer the questions on the basis of what is stated or implied in t he passages and in any introductory material that may be provided.Questions 9-12 are based on the following passages. Passage 1What accounts f or the inexorable advance of the giant sports utility vehicle (SUV) into our lives? Why do we want high-clearance trucks with 'f our-wheel drive andf ront bumpers as big as battering rams? A large part of5 the answer lies in the f ake Western names so many ofthem carry. No one much cares about what those names denote (lakes, f rontier towns, mountain ranges); whatmatters is their connotations of rugged individualism,mastery over the wilderness, cowboy endurance. The 10 names simply magnif y the appeal of these vehicles ‟that are the Frankensteinian concoctions of our private anxieties and desires., `Passage 2 'When a major manuf acturer launched an SUV namedf or an Alaskan mountain, an auto-trade publication dis-15 cussed the subtleties of its name. It proposed that eventhough most buyers will never venture into territory any less trampled than the parking lot of the local shopping mall, the important goal of the marketing hype is to plant the image in customers‟ minds that they can conquer20 rugged terrain. Perhaps we‟re trying to tame a dif ferentkind of wilderness. Indeed, in an age when many who can aff ord to do so live in limited-access communities in houses guarded by sophisticated surveillance systems, the SUV is the perfect transportation shelter to protect us 25 f rom f ears both real and imagined.9.Passage l and Passage 2 both support which of the` following generaliz ations about buyers of SUVs?(A) They intend to drive them on rough terrain.(B) They wish to live in mountainous regions.(C) They are wealthier than most other car buyers.(D) They are inf luenced by marketing strategies.(E) They are insecure about their social status.10 Which of the following aspects of SUVs is addressed in Passage l but Q in Passage 2 ?(A) Their imposing bulk (B) Their escalating cost(C) The psychology of their owners(D) Their environmental impact(E) The signif icance of their names11 Which of the following in Passage I exemplifies the “subtleties” mentioned in Passage 2, line 15 ?(A) “inexorable advance” (line 1)(B) “battering rams" (line 4)(C) “lakes, f rontier towns, mountain ranges” (line 7)(D) “connotations” (line 8)(E) “Frankenstei n ian concoctions” (line l 1)12 Passage l and the article cited in Passage 2 bothindicate that the imagery used to market SUVs is intended to(A)appeal to drivers‟ primitive instincts(B)stir yearnings f or a simpler way of li f e(C)engender f eelings of power and control(D)evoke the beauty of unspoiled nature(E) create an aura of nonconf ormity …Questions 13-24 are based on the following passages.These two passages discuss different aspects of the impact of the First World War (1914-1918) on British people and society. Passage 1 is from a book that examines the depiction of the war in literature, letters, and newspapers; Passage 2 is from a book that examines the differences between m en ’s and women 's experiences of war.Passage 1Even if the civilian population at home had wanted to know the realities of the war, they couldn‟t have withoutexperiencing them: its conditions were too novel, its indus-trialized ghastliness too unprecedented. The war would `5 have been simply unbelievable. From the very beginning af issure was opening between the amiy and the civilians.The causes of civilian incomprehension were numerous.Few soldiers wrote the truth in letters home for fear ofcausing needless uneasiness. If they did ever write the10 truth, it was excised by company off icers, who censoredall outgoing mail. The press was under rigid censorshipthroughout the war. Only correspondents willing to f ilewholesome, optimistic copy were permitted to visit France,and even they were seldom allowed near the battlef ields of15 the f ront line. Typical of these report ers was George Adam,Paris correspondent of the Times. His Behind the …Scenes atthe Front, published in 1915, exudes cheer, as well as warm condescension, toward the common British soldier, whomhe depicts as well f ed, warm, saf e, and happy-better off,20 indeed, than at home.Lord Northcliff e, the publisher of the Times, eventually assumed f ull charge of government propaganda. It is no sur- prise to f ind Northcliffe‟s Times on July 3, 1916, reportingthe f irst day‟s attack during the battle of the Somme'*with25 an airy conf idence which could not help but deepen the fdivision be tween those on the spot and those at home. “Sir …Douglas Haig telephoned last night," says the Times, “thatthe general situation was f avorable.” It soon ascends to therhetoric of heroic rom ance: “There is a f air f ield ... and30 we have elected to f ight out our quarrel with the Germansand to give them as much battle as they want.” No wondercommunication f ailed between the troopsand those whocould credit prose like that as f actual testimony. ~ ~* The British army had nearly 60.000 casualties, the largest number f or any single day in the amiy‟s history.Passage 2The First World War is a classic case of the dissonance35 between official. male-centered history and unoff icial f em alehistory. Not only did the apocalyptic events of this war have very diff erent meanings f or men and women, such eventswere in f act very different f or men and women, a pointunderstood almost at once by an involved contemporary40 like V era Brittain. She noted about her relationship withhe r soldier f iancé that the war put a “b arrier of indescribabl e experience betweensmen and written whom they loved.Sometimes (I wrote at the time) I f ear that even if he getsthrough, what he has experienced out there may change his 45 ideas and tastes utterly."The nature of the barrier thrust between V era Brittain `and her f iancé, however, may have been even more complex than she herself realized, f or the impediment preventing amarriage of their true minds was constituted not only by his 50 altered experi ence but by hers. Specif ically, as young menbecame increasingly alienat ed from their pre-war selves,increasingly immured in the muck and blood of the battle-f ields, increasingly abandoned by the civilization of whichthey had ostensibly been heirs, women seemed to become, 55 as if by some uncanny swing of history‟s pendulum. evermore powerf ul. As nurses, as munitions workers, as busdrivers, as soldiers in the agricultural “land army,” evenas wives and mothers, these f ormerly subservient creaturesbegan to loom larger. A visitor to London observed in60 1918 that; “England was a world of women-women inunif orms!"The wartime poems, stories, and memoirs by women .sometimes subtly, sometimes explicitly explore the polit-ical and economic revolution by which the First World War 65 at least temporarily dispossessed male citizens of theprimacy that had always been their birthright, while perma- nently granting women access to both the votes and theprof essions that they had never bef ore possessed. Similarly,a number of these women writers covertly or overtly cele-70 brated the release of f emal e desires and powers which thatrevolution made possible, as well as the reunion (or evenreunif ication) of women which was a consequence of such liberated energies.Their enthusiasm, which might otherwise seem like75 morbid gloating, was explained by V irginia…Woolf, a writerotherwise known f or her paci f ist sympathies:How. _ _ can we explain that amazing outburst inAugust 1914, when the daughters of educated men _ _ _rushed into hospitals. _ _ drove lorries, worked in f ields 80 and munitions f actories, and used all their immensestores of charm _ _ _ to persuade young men that to f ight was heroic. _ _ ? So prof ound was ( woman 's) unconsciousloathing f or the education of the private house that shewould undertake any task, however menial, exercise any 85 f ascination, however fatal, that enabled her to escape.Thus consciously s e desired “our splendid Empire”,'unconsciously she sired our splendid war.13 Passage 2 is unlike Passage l in that Passage 2(A) describes war as dehumanizing(B) endorses the off i cial view of the war(C). discusses war in terms of how it affects women(D) tries to identif y the root causes of the conf lict(E) criticizes the censorship of inf ormation about the war14 The “fissure” (line 6) was primarily caused by the(A) civilians‟ ignorance about the soldiers‟ experience(B) discrepancy between the experiences of men and of women(C) behavior of the off i cers who led the battles(D) guilt that civilians f elt about sending young men of f to war(E) special privileges grant ed to war correspondents15 The footnote about the battle of the Somme adds information that(A) shows how history has been rewritten to glorif y the war`(B) trivializes the dangers faced by most of the soldiers(C) emphasizes the inaccuracy of the published reports(D) suggests that the -costs of war outweighed its l benef its(E) offers a journalist‟s personal ref l ection on the war16 ln Passage l, the author suggests t hat the attitudes of “those at home” (line 26) were strongly influenced by .(A) the govern ment‟s inadequate contro l over propaganda(B) the lack of opportunities f or soldiers to write home(C) the disparity between men‟s and women‟s views of war .(D) eff orts of paci f ists to end the war(E) censored reports f rom the press17 ln line 33, “credit” most nearly means(A) award(B) believe(C) enter(D) Supply(E) enrich18. ln Passage2, the aut hor mentions V era Brittain (line40) primarily to(A) support an argument by quoting material written at the time of the war(B) present an example of the kind of powers women gained during the war ' 4 .(C) describe how a writer manipulated the f acts about the war(D) discuss the wartime literature produced by women(E) dispute recent historians‟ views of the war19 In line 58, t he reference to “wives and mothers” most directly imp lies the aut hor’s assumption that(A) families prospered more when women became head of the household(B) soldiers were unaware of the f undamental change taking place in society(C) women embraced their chance to work outside the home(D) women were anxious about f ul f illing f amily responsibilities(E) women in domestic roles had previously exercised little authority20 In line 64, t he “revolution” refers to(A)women‟s literary output during the war (B) women‟s pursuit of rights previously unavailable to them(C) the change that men underwent af ter experi encing war(D) the redistribution of power from the upper to the middle class`(E) the growing equalization of men‟s and women‟s wages21 The author of Passage2 implies that women’s enthusiasm “might . . _seem like morbid gloating” (lines 74-75) because(A) women`s progress caused the deterioration of men‟s status '(B) women achi eved recognition as the real peace- m akers in the war(C) women boasted that the war would be lost without them(D) women celebrated the fact that they did not have to f ight in the war 2(E) women were enjoying power while men were in battle22 In lines 84-85, the discussion of women’s involvement 24; with “menial” tasks and “fatal”fascinations primarily serves to emphasiz e the(A)far-reaching consequences of women‟s roles during wartime(B)extent to which women f elt stif led in their traditional roles(C)contrast between how women idealized war and what it wasreally like(D)desire by women to escape the horrors of war(E)risks that women took to f ight in the war23 What do Behind the Scenes at the Front(lines 16-17) and"wartime poems, stories and memoirs” (line 62)have in common?(A)Both caused needl ess uneasiness among civilians.(B)Both deliberately ref lected the views of the governm ent(C)Both changed the status quo f or women in wartime Britain.(D)Both encouraged writers to take their craf t more seriously.(E)Neither f ocused on the realities of the battlef i eld.24 Which of the following statements about the effect of t heFirst World War is supported by both passages?(A) Off icers resented the govern ment‟s complacency.(B)Women gained independence in postwar Britain.(C)Soldiers f elt isolated from parts of civilian society(D)Writers failed in their attempts to describe the atrocities ofwar.(E) War proved an undesirable way to resolve the Europeanconf lict.。
新通外语——SAT阅读-新SAT中美国历史如何在阅读中考察
新SAT中美国历史如何在阅读中考察(一)新SAT中,以美国历史为背景的阅读材料成为必考的内容,今天就以CB公布的样卷中的4套(OG1-4)为例子,看看新SAT美国历史相关的内容如何在阅读中进行考察。
OG 1首先我们来看第1套阅读,当中出现了很常见的关注女性权利的主题的文章,选取自1938年Virginia Woolf发表的著作《Three Guineas》。
Virginia是20世纪初英国重要的作家,也是当时著名的知识分子团体“布鲁姆斯伯里集团”(Bloomsbury Group)的核心成员。
布鲁姆斯伯里集团最初成立时有点像Cambridge同学会,一帮玩的好的朋友聚在一起吃吃喝喝。
但是由于后来大名鼎鼎的经济学家Keynes和诗人T. S. Elliot当时都是其座上宾,因此近些年引起了历史学家的注意,他们对之后艺术、文学和女权主义等方面产生了巨大的影响。
Virginia在1929年出版《A Room of One's Own》, Virginia的名言“A woman must have money and a room of her own if she is to write fiction”正是源于此。
1938年出版了阅读中节选的Three Guineas,对女性地位的提升和权益的保护大声疾呼。
关于女权主义的相关内容我们在之前的讲座中有提到过。
与欧洲一样,美国女性的地位一直以来都非常低,1848年Seneca Falls Convention的召开标志着人们、尤其是女性对女性自身权利的关注,一般也认为其标志着女权主义的诞生。
之后经过漫长的发展,一直到20世纪初,英国和美国才开始向女性开放政治选举。
接着后面两篇文章关注的是太空矿产开采的问题,其中也涉及到“淘金热”和资本垄断等问题,这些在美国历史中都是非常重要的议题。
OG 2第2套阅读中33-42题的阅读文章继续第1套对女性的关注,选取了19世纪中叶美国女权主义重要代表人物Elizabeth Stanton的1869年在华盛顿举行的Woman Suffrage Convention上的演讲作为阅读材料。
SAT阅读黑人文学常见主题(一)
SAT阅读黑人文学常见主题(一)
在SAT阅读中,所考的文章选段大多从文学名著中直接节选或改编。
涉及题材较广,如学生考前略知一二则对考试大有裨益。
本文就来列举一些黑人文化中常考的话题。
黑奴贸易(slave trade)是必谈的部分,欧洲走向了殖民掠夺的道路。
在美洲,欧洲殖民者(colonist)强占印第安人的土地建立庄园(plantation),同时,肆意屠杀和奴役印第安人。
在印第安人(Native Americans)数量锐减的时候,种植园主开始了买进非洲黑人,作为奴隶在庄园里劳动,这就是欧洲贩卖黑人的开始。
黑奴贸易(slave trade)加速了欧洲的资本积累(accumulation of capital),也为美洲地区的开发提供了劳动力。
19 时间30 年代初在美国北部兴起了废除黑人奴隶(abolition of slavery)的运动,各种反奴隶制协会在北部建立,并积极开展各种活动。
但是废奴运动(abolition movement )进行的并不顺利,因为南方的种植业(planting)迅速发展,种植园奴隶制(slavery)不易被推倒。
最终,南北战争的爆发使得至少6 万的黑奴获得自由。
在整个的废奴过程中涌现出了大量的文学作品,其中包括著名的《汤姆叔叔的小屋》(Uncle Tom’s Cabin)。
同学们在读到此类的文学文章时应注意他的时代背景,以及作者的写作目的。
文本提供:/ncsat/。
SAT阅读必备OG之Victoria时期中产阶级妇女的经历
SAT阅读必备OG之Victoria时期中产阶级妇女的经历今天文都国际小编为大家分享的是SAT阅读必备OG之Victoria时期中产阶级妇女的经历,方便大家学习整理。
希望对同学们有帮助。
P578——Section 2Victoria时期中产阶级妇女的经历1、ANSWERS AND EXPLANATIONSExplanation for Correct Answer C :Choice (C) is correct. "Occupation" in this context refers to a "vocation," or suitable work. If one were to insert this definition into the sentence would read: "Thus, at a time when vocation was becoming a core element in masculine identity, any position for middle-class women other than in relation to men was considered anomalous."Explanation for Incorrect Answer A :Choice (A) is incorrect. In a military context, "occupation" refers to the control of a nation by foreign forces. The passage does not mention the military at all, however, so this definition is inappropriate.Explanation for Incorrect Answer B :Choice (B) is incorrect. "Occupation" can mean a hobby or diversion, but this passage clearly refers to the working world and not to leisure activities.Explanation for Incorrect Answer E :Choice (E) is incorrect. While it may be inferred that Victorian women were victims of political as well as social repression, "occupation" does not logically signify any sort of repression within the context.2、ANSWERS AND EXPLANATIONSExplanation for Correct Answer E :Choice (E) is correct. According to Passage 1, a "fifth class" was created in the nineteenth century to describe the large numbers of middle-class women who did not work outside the home. The existence ofsuch a class contrasts sharply with the social climate of the seventeenth century, when women played a significant role in family businesses, as evidenced by the trade tokens that carried their initials.Explanation for Incorrect Answer A :Choice (A) is incorrect. Passage 1 does not address whether trade tokens qualified as legal currency. Regardless, the monetary value of these tokens is irrelevant in a discussion of the societal status of working women.Explanation for Incorrect Answer B :Choice (B) is incorrect. Although trade tokens may have been issued to women of different classes, the author considers the initials on the tokens and not the tokens themselves to be evidence of prevailing attitudes of the seventeenth century.Explanation for Incorrect Answer C :Choice (C) is incorrect. The author of Passage 1 does not indicate that the trade tokens had any effect on gender stereotypes.3、ANSWERS AND EXPLANATIONSExplanation for Correct Answer D :Choice (D) is correct . Queen Victoria does not reflect the "diminished social status" of Victorian women. Rather, she appears in the passage as a marked exception to the rule.Explanation for Incorrect Answer A :Choice (A) is incorrect. Passage 1 makes several allusions to the disparity that existed in the workplace between Victorian men and women, asserting that "inequality in the working world made it exceedingly difficult for a middle-class woman to support herself."Explanation for Incorrect Answer C :Choice (C) is incorrect. Passage 1 mentions that by the end of the eighteenth century, women's initials were no longer retained on familytrade tokens. This detail indicates that women were no longer regarded as significant contributors to family businesses.Explanation for Incorrect Answer E :Choice (E) is incorrect. Passage 1 asserts that the absence of women's financial documents from the nineteenth century illustrates the degree to which they disappeared from business affairs.4、ANSWERS AND EXPLANATIONSExplanation for Incorrect Answer A :Choice (A) is incorrect. While it may be true that a seventeenth-century woman worker's status was enhanced by her responsibilities, these lines are concerned with only women of the eighteenth and nineteenth centuries.Explanation for Incorrect Answer E :Choice (E) is incorrect. Formal academic institutions may not have admitted women in the seventeenth century, but this claim does not support the view that workplace opportunities for women decreased between the eighteenth and nineteenth centuries.5、ANSWERS AND EXPLANATIONSExplanation for Correct Answer D :Choice (D) is correct. "Hail" in this context means to welcome, or to greet.It comes from Davenport Adams's assertion that it is natural that a woman who is "fettered," or repressed, by Victorian society should welcome the emancipation, or freedom, that travel can provide.Explanation for Incorrect Answer A :Choice (A) is incorrect. It is illogical to say that women should "call out to" a concept, such as emancipation.Explanation for Incorrect Answer B :Choice (B) is incorrect. This definition of "hail" is inappropriate, given the context of line 80. Freedom cannot be "hailed" with a physicalgesture.Explanation for Incorrect Answer C :Choice (C) is incorrect. It does not make sense to say that Victorian women "should come from" freedom when, in fact, they are not free at home.Explanation for Incorrect Answer E :Choice (E) is incorrect. "Hail" does not mean to summon, or to call for, in this context.6、ANSWERS AND EXPLANATIONSExplanation for Correct Answer E :Choice (E) is correct. The passage indicates that Kingsley's attitude toward women's rights campaigns was one of distaste, despite the fact that her travels identified her as a liberated, "new woman."Explanation for Incorrect Answer A :Choice (A) is incorrect. Kingsley was a traveler, not an activist. According to the passage, Kingsley was "chagrined" to learn that she had become a symbol of "the new social and political freedom and prowess of women."Explanation for Incorrect Answer B :Choice (B) is incorrect. While Kingsley may have felt a degree of antagonism toward those who pressured her to become a spokeswoman for a movement that she did not identify with, there is nothing to suggest that dedication to another cause prompted the hostility.Explanation for Incorrect Answer D :Choice (D) is incorrect. Kingsley's attitude of distaste toward women's rights campaigns suggests that she was either uninterested in the movement or was simply opposed to women's struggle for freedom. The passage does not mention other groups.7、ANSWERS AND EXPLANATIONSExplanation for Incorrect Answer A :Choice (A) is incorrect. This option neglects the fact that women also traveled for humanitarian purposes.Explanation for Incorrect Answer B :Choice (B) is incorrect. The passage says nothing to suggest that women traveled to pursue business-related interests.Explanation for Incorrect Answer D :Choice (D) is incorrect. According to Passage 2, women often traveled as missionaries. There is nothing in the text to suggest that their trips were business-related, however.Explanation for Incorrect Answer E :Choice (E) is incorrect. Passage 2 specifies that British women traveled for educational reasons, but it says nothing about their entrepreneurial pursuits.8、 ANSWERS AND EXPLANATIONSExplanation for Incorrect Answer B :Choice (B) is incorrect. Passage 2 describes the "autonomy" that middle-class women find in their travels; an aristocrat living abroad with her father is neither middle-class nor independent.Explanation for Incorrect Answer C :Choice (C) is incorrect. Passage 2 mentions women who travel alone as missionaries to "escape domestic confinement," but a woman who relocates abroad with her husband is neither traveling alone nor escaping her housebound duties.Explanation for Incorrect Answer D :Choice (D) is incorrect. A nursemaid is dependent on her employers and therefore falls outside the scope of Passage 2's argument, which centers on middle-class women traveling to achieve independence.Explanation for Incorrect Answer E :Choice (E) is incorrect. A girl from a poor family who is sent abroadto work is neither middle-class nor an independent traveler.9、ANSWERS AND EXPLANATIONSExplanation for Correct Answer B :Choice (B) is correct. The "fifth class" described in Passage 1 consists of women confined to household activities. Passage 2 refers to these housebound women as "caged birds."Explanation for Incorrect Answer A :Choice (A) is incorrect. Female missionaries who worked outside the home and out of England would certainly not have qualified as members of the "fifth class."Explanation for Incorrect Answer C :Choice (C) is incorrect. The "new woman" described in Passage 2 traveled and, therefore, was not bound by the household duties that defined the "fifth class."Explanation for Incorrect Answer D :Choice (D) is incorrect. Middleton is quoted as an author, and Kingsley was an independent traveler; neither is representative of the Victorian "fifth class."Explanation for Incorrect Answer E :Choice (E) is incorrect. Fussell's and Adams's first names indicate that they are both men, and are thus disqualified from membership in the "fifth class," which is "exclusively made up of women."10、ANSWERS AND EXPLANATIONSExplanation for Correct Answer B :Choice (B) is correct. The tone of both passages can be described as objective and unemotional.Explanation for Incorrect Answer C :Choice (C) is incorrect. Neither passage expresses the personal feelings, regretful or otherwise, of its author.Explanation for Incorrect Answer D :Choice (D) is incorrect. Although Passage 1 discusses the inequality that Victorian women faced, the author remains objective. Additionally, neither passage expresses righteous indignation, or justified anger.Explanation for Incorrect Answer E :Choice (E) is incorrect. There is not a trace of hostility in the tone of either passage.以上就是SAT阅读必备OG及答案解析。
satog5阅读真题解析
SAT test 51. For a long time, most doctors maintained that taking massive doses of vitamins was relatively harmless; now, however, some are warning that excessive dosages can be _________.(A) healthy adj. 健康的(B) expensive adj. 昂贵的(C) wasteful adj. 浪费的(D) toxic adj. 有毒的(E) inane adj. 愚蠢的解析:D,在很长的一段时间里,大多数医生认为用大量的维他命是无害的;不过现在有些人警告过量食用会----。
这里however表示转折与前面的harmless是相反的意思。
2. In Jamaica Kincaid's novel Lucy, the west Indian heroine _________ her employers' world, critically examining its assumptions and values.(A) idealizes v. 理想化(B) avoids v. 避开(C) beautifies v. 美化(D) scrutinizes v. 仔细检查(E) excludes v. 排除,解析:E,在牙买加金彩的小说露西,这个西印度群岛英雄----她的雇主的世界,精细的审视他的猜测和价值。
这个句子中前后两句意思是一致的,没有转折词,所以空格所需的东西与examining是同义词,所以D。
frequent name changes that the country has undergone _________ the political turbulence that has attended its recent history.(A) argue against v. 真钞,辩论(B) contrast with v. 对比,差异(C) testify to v. 证明(D) jeopardize v. 危及(E) sustain v. 支撑解析:C,这个国家经历了频繁的国名变更---这个国家的近点史上的政治动乱。
SAT阅读必备OG之女性作家面临的挑战
SAT阅读必备OG之女性作家面临的挑战今天文都国际小编为大家分享的是SAT阅读必备OG之女性作家面临的挑战,方便大家学习整理。
希望对同学们有帮助。
P973——Section 6女性作家面临的挑战1. ANSWERS AND EXPLANATIONSExplanation for Correct Answer D :Choice (D) is correct. Douglass evoked the highly respected concept of individualism, identifying himself "with the triumph of manliness and individualism that slavery suppressed". His writings showed how the conditions of slavery directly undermined this basic human ideal.Explanation for Incorrect Answer A :Choice (A) is incorrect. Douglass "firmly identified himself" with ideas of manliness and individuality, concepts popular with middle-class, northern Whites. Douglass did not reject these notions. Rather, he relied on them to reveal the injustice of slavery.Explanation for Incorrect Answer B :Choice (B) is incorrect. The passage does not argue that Douglass’s writings laid out a specific criticism of Whites or a strategy to resolve their wrongdoing. Instead, he used Whites' ideas of humanity and freedom to show them the injustice of slavery.Explanation for Incorrect Answer C :Choice (C) is incorrect because Douglass did not draw on unusual or unfamiliar material to make his case against slavery. Rather, he evoked concepts familiar to his White readers.Explanation for Incorrect Answer E :Choice (E) is incorrect. The passage does not indicate that Douglass’s writings drew on one particular event. Douglass referred to popularbeliefs of the day to show Whites how slavery violated their own deeply held principles.2. ANSWERS AND EXPLANATIONSExplanation for Correct Answer D :Choice (D) is correct. To "formulate" is to "work out" or develop an idea. This is what a small number of northern White women were beginning to do regarding the oppression of women and slavery: "work out" the analogy between them.Explanation for Incorrect Answer A :Choice (A) is incorrect. To "exercise" means to put into action. The passage implies that the "analogy between slavery and the oppression of women" was still being developed; it was too early to put it into action.Explanation for Incorrect Answer B :Choice (B) is incorrect. To "conciliate," which means to win over or soothe the anger of another, is unrelated to the early development of an idea.Explanation for Incorrect Answer C :Choice (C) is incorrect. In this context, to "work out" means to sketch out preliminary ideas or develop an idea. It does not mean to struggle, or make an effort, for change.Explanation for Incorrect Answer E :Choice (E) is incorrect. It does not make sense to say that the women were beginning to "solve" the analogy between slavery and their own oppression. An analogy is not a puzzle to be solved; it is a statement of the similarities between two things.3. ANSWERS AND EXPLANATIONSExplanation for Correct Answer B :Choice (B) is correct. The passage explains that women who wanted to gain rights "clung to the discourses of true womanhood and domesticity".Those discourses represented the view of women with which most people were sympathetic, that women were first and foremost mothers and homemakers. Thus, arguing for rights on the grounds of improving women's ability as mothers would fit the passage's claim precisely.Explanation for Incorrect Answer A :Choice (A) is incorrect. This statement does not build on the notion of women as essentially domestic creatures (mothers and homemakers), which was the prevailing view of women at the time. According to the passage, therefore, this argument would not have been an effective means of gaining support for women’s rights.Explanation for Incorrect Answer C :Choice (C) is incorrect. The passage explains that women who wanted to gain rights "clung to the discourses of true womanhood and domesticity" . In the pre-Civil War era, the prevailing view was that it was women’s responsibility to be productive in the home, not in society at large. Therefore, this argument would not have been an effective means of gaining support for women’s rights.Explanation for Incorrect Answer D :Choice (D) is incorrect. As the passage states, women "were beginning to work out the analogy between slavery and the oppression of women, but their view had not won general sympathy". Women who wanted to gain rights "clung to the discourses of true womanhood and domesticity" , not those of the anti-slavery movement.Explanation for Incorrect Answer E :Choice (E) is incorrect. At the time, self-determination, or free will, was not considered an important aspect of women’s nature. Women who wanted to gain rights "clung to the discourses of true womanhood and domesticity" , not those of individualism.4. ANSWERS AND EXPLANATIONSExplanation for Correct Answer A :Choice (A) is correct. The passage describes in detail how Jacobs "shaped her presentation of herself to conform" to audience expectations . Jacobs carefully framed her story to achieve the end result of reaching a certain audience. It thus makes sense to describe her as "pragmatic," or concerned with results.Explanation for Incorrect Answer B :Choice (B) is incorrect. "Disingenuous" means not straightforward, or dishonest. Nothing in the passage indicates that Jacobs relied on false information or half-truths to communicate her story.Explanation for Incorrect Answer C :Choice (C) is incorrect. "Scholarly" writing is academic, or written for a highly educated reader. The passage argues that, on the contrary, Jacobs addressed her writing to a wide audience, "reaching the same people who avidly read Harriet Beecher Stowe," a popular novelist.Explanation for Incorrect Answer D :Choice (D) is incorrect. "Presumptuous" means overly confident or arrogant. Jacobs did not presumptuously demand that her audience interpret her writing; she adjusted her writing to suit the needs of her audience.Explanation for Incorrect Answer E :Choice (E) is incorrect. Nothing in the passage indicates that Jacobs wrote in a "melodramatic," or exaggeratedly emotional or sentimental, manner.以上就是SAT阅读必备OG及答案解析。
SAT阅读必备OG之美洲黑人与非洲黑人篇
SAT阅读必备OG之美洲黑人与非洲黑人篇今天文都国际小编为大家分享的是SAT阅读必备OG之美洲黑人与非洲黑人的关系,方便大家学习整理。
希望对同学们有帮助。
美洲黑人与非洲黑人的关系1、ANSWERS AND EXPLANATIONSExplanation for Correct Answer B :Choice (B) is correct. Line 5-7, ancestor were sending…to descendants,并且两个形容词clear and powerful,可以排除A选项的veiled, B选项的questionable,只有B选项的cautionary,劝诫的,告诫性的,对应的最好。
2、ANSWERS AND EXPLANATIONSExplanation for Correct Answer E :Choice (E) is correct. line41-42, shodoway imagings 和real experience相对应,既然是imaging的,并且和real相反,所以是E选项的unsubstantiated,没有证据的,未被证实的。
3、ANSWERS AND EXPLANATIONSExplanation for Correct Answer E:Choice (E) is correct. line 42, disappointed,以及line 43-44,三个if所体现出来的语气可以知道应该是负态度,而line 46,But……the world has been forced to take note,说明作者为非洲黑人与美洲黑人受全世界关注而感到自豪,所以E is correct.4、ANSWERS AND EXPLANATIONSExplanation for Correct Answer D :Choice (D) is correct. 文章引言”Here, he offers his views on the historical relationship between Black Americans and Black Africans”,暗示全文讲述的是两者之间的relationship,对应D选项和E选项,而E选项的"cultrual ties”范围太小,仅仅只是文化的关系,而全文讲述了Black Americans and Black Africans的身份认同问题,文化只是一方面而已。
nasa黑人女科学家英文阅读理解
nasa黑人女科学家英文阅读理解Title: The Journey of a NASA Black Female Scientist.In the vast and ever-expanding universe, one woman stands out as a beacon of inspiration and perseverance. Dr. Katherine Johnson, a black female scientist at NASA, has dedicated her life to the exploration of space and the advancement of science. Her remarkable journey is a testament to the power of determination and the beauty of breaking barriers.Born in White Sulphur Springs, Virginia, in 1918, Katherine Johnson faced numerous challenges throughout her childhood. She was raised in a segregated society where opportunities for black women were limited. Despite these obstacles, she possessed a natural curiosity about the world and a desire to learn. Her parents recognized her intelligence and encouraged her to pursue her dreams, instilling in her the belief that anything was possible with hard work and perseverance.Johnson's interest in mathematics and science grew as she attended high school. She excelled in her studies and was offered a scholarship to attend West Virginia State College, where she majored in mathematics and physics. During her time at college, she faced discrimination and racism, but she persevered and graduated with honors in 1937.After college, Johnson's career path led her to NASA, where she became a key figure in the space race. Initially, she was hired as a "computer" at the agency's Langley Research Center, performing complex mathematicalcalculations by hand. Her work was crucial in the development of the orbital mechanics needed to send astronauts into space.Despite the racial discrimination she faced at NASA, Johnson's talent and dedication to her work were recognized. She was promoted to become one of the agency's first black female engineers and was instrumental in the success of missions like John Glenn's orbit of Earth in 1962. Hercalculations were vital in ensuring the safety and accuracy of these missions, and her contributions were recognized by both NASA and the scientific community.Johnson's legacy is not only in the scientific advancements she made but also in the inspiration she imparts to generations of young people. Her story is a reminder that anyone, regardless of their background or challenges, can achieve their dreams with hard work and determination. She has broken barriers and paved the wayfor other black women in science and engineering, showing that the sky is not the limit when it comes to achieving success.In conclusion, the journey of Dr. Katherine Johnson is an inspirational tale of perseverance and success. She has overcome racial discrimination and personal challenges to become a pioneering scientist at NASA, making significant contributions to the field of space exploration. Her story reminds us that we should never underestimate the power of our dreams and the impact we can make in the world. As we look to the stars and the future of space exploration, itis important to remember the legacy of Katherine Johnson and the countless other black women scientists who have broken barriers and made history in the field of science.。
SAT阅读考试文章特点剖析
SAT阅读考试文章特点剖析
SAT阅读文章的分类:
依据文章体裁:non-literary / literary fiction
依据文章长度:short passage / long passage
依据文章数量:single passage / paired passages
SAT阅读文章的题材特点特别鲜亮,一共包括了下面几个类别。
移民文化 (cross-culture and emigration)
黑人土著 (Black Americans Native Americans)
女性女权 (women feminism)
生物环境 (biology environment)
自然科学 (natural science)
文学作品 (literary fiction)
艺术评论 (art criticism)
社会讨论 (social studies)
排列组合之后考试时所见到的SAT阅读文章类型有:
SSP (short single passage)
SPP (short paired passages)
LSP (long single passage) (non-literary)
LF (literary fiction)
LPP (long paired passages)
SAT阅读文章中各类题型:推理细节态度词汇作用例子主旨互联求同求异修辞外援符号
SAT阅读文章的备考中需要大家关注的学问点是:source, time, background, author (status), keywords, theme, etc.
以上便是SAT阅读考试文章特点剖析的相关介绍,特别具体,盼望对大家有所帮忙。
SAT_OG_阅读翻译
SAT OG 阅读译文译者:黄晓红SAT Practice Test #1section2p391【图:Kitt Peak National Observatory 美国国家天文观测台】(此刻,我站在基特峰1的最高处)一个个的望远镜天文台伫立在基特峰上,风从它们中间呼啸而过。
下方,连绵起伏的云被黄昏染成了灰色,它们仿佛汇聚成一条长河,缓缓流过。
夜空中,海尔-波普彗星恰似一个羽毛状的鱼饵,高高地悬挂在天幕上。
它的尾巴略微有点弯,就好像被强劲的风吹到了一边。
群星点缀着深沉的夜空,一闪一闪地眨着眼睛。
几只漫步的野马从我身旁走过。
然而,它们不曾抬头仰视天空中那薄纱似的彗星,或是在某个清朗的晚上(不论是否有彗星),一瞥这神秘而美丽的夜空奇观。
这样的感觉真好。
6.第12行中,作者暗示作为“人”意味着(A)偶尔犯错(B)喜欢他人的陪伴(C)思索过去的经历(D)欣赏自然之美(E)从简单中寻找乐趣7.这篇文章中,起重要作用的修辞手法是(A)唤起情绪(B)比喻性的语言(C)类推(D)倒叙(E)讽刺1备注:美国国家天文观测台地址Augusta Ada King1843年,艾达·金发表了一批颇具影响力的笔记,阐述巴贝奇关于“分析机”——第一台自动化计算机的设计——的构想。
金的笔记,其中还包括了她用来计算一系列叫“伯努利数”的程序,确立了她在计算机科学领域的重要地位。
然而,她令人称奇的一生和家世(她是大名鼎鼎的诗人拜伦的女儿),以及她在自身研究领域中女性开拓者的身份,使她变成了一个偶像。
她给传记、戏剧、小说,甚至还有一部故事片,提供了灵感。
尽管很多女性都推动了计算机科学的进步,唯独金获此殊遇:有一门计算机语言以她命名:Ada。
8.这篇文章主要是关于(A)解释艾达对计算机科学的兴趣(B)提供关于艾达的人格分析(C)概述艾达如何有名以及为何有名(D)追溯当代计算机的发展历程(E)鼓励更多女性投身计算机科学领域9.关于艾达·金,本文作者最有可能不同意一下哪项?(A)她的家世与她受到的关注没有关系(B)她对计算机科学的贡献是原创性的(C)对她的兴趣蔓延到流行文化中(D)在她完成工作后的很长一段时间里都被人熟知(E)即使不考虑她对计算机科学的贡献,她也是不同寻常的p392Famous African Americans下面这篇文章改编自一部1999年的回忆录。
黑人女性寻找自我的艰辛
黑人女性寻找自我的艰辛【摘要】托尼·莫里森的小说《秀拉》为我们塑造了三位鲜活的女性人物形象:秀拉、奈尔和爱娃,让我们看到黑人女性多姿多彩的一面。
本文着重分析这三位女性的性格及其意义。
【关键词】托尼·莫里森秀拉黑人女性《秀拉》,是美国1993年诺贝尔文学奖得主托尼·莫里森的一篇较有影响的小说。
在这篇小说中,莫里森为我们刻画了众多的人物形象,他们给读者留下了深刻的印象。
秀拉,作为小说中的同名主人公更是引人注目。
作者把秀拉比作是一支带刺的玫瑰花,书前题字更是一语中的:“世人无人曾知晓我的玫瑰,除去我自己……我有过极大的荣誉。
别人却在内心里不需要那样的荣誉。
”〔2〕(136)的确,在作品中,作者赋予秀拉太多的荣光的同时,也使她经历了无尽的艰辛与磨难。
秀拉是一个敢做敢为、意志坚定的反传统女性,她不懈地发掘自我,解放自我,踏上了一条人迹罕见、崎岖不平的泥泞的小路。
这条小路也许几十年、几百年之后会变成通渠大道,但秀拉却无缘看到追寻她的足迹的后来者,因为远在故事结束以前的1941年(小说的结尾是1965年)秀拉就已经去世了。
但她留给我们的思索却绵延到现在,以至更远……秀拉的外婆伊娃是一个刚烈、强悍、极端、坚毅的女人;汉娜则是一个温柔、敏感的女人,而秀拉则是综合了她们两个性格中的全部,并把它们发挥得淋漓尽致,秀拉比她的两个前辈走得更远。
伊娃是这个家庭的缔造者。
她是一个尽职尽责、富有牺牲精神的母亲。
丈夫鲍伊鲍伊抛弃了她,给她留下一贫如洗的家和三个嗷嗷待哺的孩子。
伊娃试图独自抚养孩子,但尽管有邻居的接济,她还是感到无能为力。
终于有一天她把三个孩子托付给邻居,自己离家出走了。
一年半后,她回来了,拄着双拐,少了一条腿,但却带回来不少钱。
为了三个年幼的孩子,她付出了多大的代价,做出何等的牺牲。
后来在1923年的夏天,当她看到汉娜在院子里不慎引火烧身的时候,不顾一切跳楼而出,她自己摔得鲜血淋淋。
- 1、下载文档前请自行甄别文档内容的完整性,平台不提供额外的编辑、内容补充、找答案等附加服务。
- 2、"仅部分预览"的文档,不可在线预览部分如存在完整性等问题,可反馈申请退款(可完整预览的文档不适用该条件!)。
- 3、如文档侵犯您的权益,请联系客服反馈,我们会尽快为您处理(人工客服工作时间:9:00-18:30)。
SAT阅读必备OG之黑人女性今天文都国际小编为大家分享的是SAT阅读必备OG之黑人女性,方便大家学习整理。
希望对同学们有帮助。
P911——Section 6黑人女性1. ANSWERS AND EXPLANATIONSExplanation for Correct Answer A :Choice (A) is correct. "To assert" means to state or declare. Present a statement or declaration of the authors' position: "But no matter what anyone may say to the contrary, Black women are different."Explanation for Incorrect Answer C :Choice (C) is incorrect. A decision resolves uncertainty, but there is no uncertainty present. The authors do not seem to doubt that "the story of Black women was a remarkable one" and that it should be told separately from the histories of African Americans and of women. Their statement that "Black women are different" is not a decision. It is simply an assertion of their position.Explanation for Incorrect Answer D :Choice (D) is incorrect. A concession is an acknowledgment that someone else's point is correct. The sentencedoes contain an acknowledgment that others hold a different point of view ("But no matter what anyone may say to the contrary"), but it does not concede that they are right. In fact, the sentence as a whole flatly contradicts the view presented at the end of the previous paragraph.Explanation for Incorrect Answer E :Choice (E) is incorrect. Although the authors of the passage do seem to criticize some historians' views at other points in the text, this is not the primary purpose. The primary purpose is to forcefully state theauthors' position (that "no matter what anyone may say to the contrary, Black women are different").2. ANSWERS AND EXPLANATIONSExplanation for Incorrect Answer A :Choice (A) is incorrect. The authors describe Black women's perspective as the result of a combination of their identities as women and as African Americans, "not just a mixture" of those identities. This perspective, the passage suggests, is the result of challenges that are unique to Black women. But the chemistry metaphor does not help characterize those challenges.Explanation for Incorrect Answer B :Choice (B) is incorrect. Since the passage does not mention scientific studies of race, class, or gender, it does not make sense to say that the passage uses the chemistry metaphor to illustrate connections between such studies.Explanation for Incorrect Answer C :Choice (C) is incorrect. The passage does not mention any "agents for change in a particular social transition." It is concerned with making the point that the experiences of Black women—what they "have experienced and still experience today"—result from a unique interaction of race and gender. The chemistry metaphor serves to emphasize the idea that "after you've told the story of African Americans and the story of women" there is still a further story to tell that is not reducible to these other stories.Explanation for Incorrect Answer E :Choice (E) is incorrect. The passage does not suggest that racism and sexism are similar. In particular, the chemistry metaphor does nothing to show any similarities between them.3. ANSWERS AND EXPLANATIONSExplanation for Correct Answer B :Choice (B) is correct. The authors say that many Black men and many White feminists "want to put their arms metaphorically around the Black woman's shoulder and say, 'She's with us.'" The authors add that, although Black women are sought after by both these groups, "they will be valued for their difference so long as they do not mention it too often" . The suggestion is that both groups see an advantage in claiming Black women as allies, but only as long as Black women can be kept from insisting on having the uniqueness of their experiences recognized. According to the authors,thefigurative gesture toward Black women is both self-interested and calculating.Explanation for Incorrect Answer A :Choice (A) is incorrect. The authors say that both Black men and White feminists "want to put their arms metaphorically around Black women's shoulders" . But this figurative gesture is presented as a way of claiming Black women for their own cause, not a gesture of affection for Black women themselves.Explanation for Incorrect Answer C :Choice (C) is incorrect. Genuine empathy involves being aware of and sensitive to the feelings of others. But according to the authors, Black women "will be valued for their difference so long as they do not mention it too often". This suggests that Black women's value in this context is dependent on their ability to keep their mouths shut. In claiming Black women for their respective groups by putting "their arms metaphorically around the Black woman's shoulder" , neither Black men nor White feminists are being genuinely empathetic.Explanation for Incorrect Answer D :Choice (D) is incorrect. There is no indication in the passage that either Black men or White feminists require any special courage to claimBlack women for their respective groups. The suggestion is rather that both groups claim Black women out of self-interest.4. ANSWERS AND EXPLANATIONSExplanation for Correct Answer D :Choice (D) is correct. The "sad situation" is that having to acknowledge difference "makes people feel nervous and guilty". The authors speculate that "maybe it is the American obsession with race that makes it so difficult for us as a nation to get rid of our fear that difference implies, even guarantees, animosity and opposition". This suggests that the explanation for the "sad situation" is people's fear that focusing on differences will lead to disharmony.Explanation for Incorrect Answer A :Choice (A) is incorrect. The "sad situation" is that having to acknowledge difference "makes people feel nervous and guilty". The authors suggest that the explanation for the "sad situation" is people's fear that focusing on differences will lead to disharmony. Further, two of the groups discussed in the passage, Black political leaders and White feminist political leaders, are specifically working for political change. Thus it does not make sense to describe resistance to political change as the explanation for the "sad situation" mentioned.Explanation for Incorrect Answer B :Choice (B) is incorrect. The passage does not mention role models or people's fears of being held up as role models.Explanation for Incorrect Answer E :Choice (E) is incorrect. The "sad situation" is that having to acknowledge difference "makes people feel nervous and guilty". The authors suggest that the explanation for the "sad situation" is people's fear that focusing on differences will lead to disharmony. This explanation depends on conformity in a specific respect: people beingreluctant to acknowledge difference. Thus placing a high value on nonconformity—that is, on being different from other people—cannot explain the "sad situation."以上就是SAT阅读必备OG及答案解析。